Középiskolai Matematikai és Fizikai Lapok
Informatika rovattal
Kiadja a MATFUND Alapítvány
Már regisztráltál?
Új vendég vagy?

Fórum: GEOMETRIA

  [1]    [2]    [3]    [4]    [5]    [6]    [7]    [8]    [9]    [10]  

Szeretnél hozzászólni? Jelentkezz be.
[1141] BohnerGéza2008-10-31 00:06:46

A zalaegerszegi Zrínyi Miklós Gimnázium "IZSÁK IMRE GYULA" komplex természettudományi versenyének egyik szép feladatát ajánlom.

141. feladat: A k1 és k2 kör két pontban, A-ban és B-ben metszik egymást. Legyen C a k1-nek k2-n kívüli olyan pontja, melyre a CA egyenes k2-t olyan az A-tól különböző D pontban metszi, mely nincs a k1-en belül. Továbbá legyen K a CD, M a k1 A-t nem tartalmazó BC ívének és N a k2 A-t nem tartalmazó DB ívének felezőpontja. Bizonyítandó, hogy MKN szög derék!

[1140] HoA2008-10-26 23:06:36

Pedagógiailag nagyon jó feladatnak tartom. Szinte vonzza a szemet, hogy AC felezőpontját E és G között NEM jelöli az F betü. Honnan ismerős a szakaszt két oldalról érintő két kör, ráadásul az ábra alapján a szakasz felezőpontjára szimmetrikus két pontban? Először belátjuk, hogy ez nem csak ránézésre van így, hanem a [1139] - beli egyenlőség éppen a feladat feltételeiből következik. És ami még "hiányzik" az ábráról: ABC beírt k1 körének E-vel átellenes H pontja - és ugyanígy ACD beírt k2 körének G-vel átellenes I pontja , melyek úgy tűnik, rajta vannak BG-n illetve DE-n. Végül az ABC háromszög AC oldalát G-ben érintő hozzáírt k3 kört megrajzolva teljes az ábra. Így már jól láthatóak a beírt és hozzáírt körök vizsgálatánál megismert hasonlóságok. k1 és k3 külső hasonlósági pontja B, ezért B, H és G valóban egy egyenesen van. k1 és k2 külső hasonlósági pontja P, ezért P, G és H egy egyenesen van. Vagyis P, G, H, B egy egyenes pontjai. És persze ugyanez áll P, D, I, E -re.

Előzmény: [1134] BohnerGéza, 2008-10-12 01:13:22
[1139] BohnerGéza2008-10-26 07:49:22

Segítség a 140. feladathoz: Bizonyítsuk, hogy AG=CE!

( A feladat szövegében természetesen a BDA-é helyett CDA-é a helyes, ahogy az ábra is mutatja. )

Előzmény: [1134] BohnerGéza, 2008-10-12 01:13:22
[1138] HoA2008-10-20 08:50:44

Két előzetes megjegyzés:

1) Nem nyilvánvaló kapcsolatot találni egy ábra bizonyos pontjain átmenő körök középpontjai és az ábra többi része között. Az itt célravezető meglátás: két egymást metsző kör centrálisa merőleges a közös húrra. Ezt a bizonyításban többször is felhasználjuk.

2) Ha sikerül bebizonyítnai, hogy tetszőleges három körülírtkör-középpont a körülírt körök közös M pontjával egy körön van, akkor egyúttal azt is bizonyítottuk, hogy a négy körülírt kör középpontja és M egy körön van, hiszen például ha O1,O2,O3ésM egy körön van , valamint O1,O2,O4ésM egy körön van, akkor ez úgy is fogalmazható, hogy O3ésO4 is rajta van az O1,O2 és M ponthármas által meghatározott körön.

[1136] ábráját megbetűzve MF merőleges O1O2 -re, ME merőleges O1O3 -ra , így nem egymás szögtartományába eső merőleges szárú szögek lévén FME\angle=O2O1O3\angle. Hasonlóan adódik, hogy EMC\angle=O1O3O2\angle. A kettőt összegezve FMC\angleO3O2O1\angle kiegészítő szögével egyenlő.

Jelöljük az MEF\angle -et \phi -vel. Az O1 középpontú k1 körben ez az MF húrhoz tartozó kerületi szög. MFO1 egyenlőszárú \Delta -ben \phi a középponti szög, MO1F\angle fele, így FMO1\angle=\pi/2-\phi.

Az O3 középpontú k3 körben CEM\angle=\pi-\phi a rövidebbik CM ívhez tartozó kerületi szög, a hosszabbik CM ívhez tartozó kerületi szög \phi, az MCO3 egyenlőszárú \Delta -ben \phi a középponti szög, MO3C\angle fele, így CMO3\angle=\pi/2-\phi.

O1MO3\angle=FMC\angle-FMO1\angle+CMO3\angle=FMC\angle, az O3O2O1\angle kiegészítő szöge, tehát O1,O2,O3ésM egy körön vannak.

Előzmény: [1136] BohnerGéza, 2008-10-16 19:03:53
[1137] HoA2008-10-17 11:54:55

Nem feledkeztem meg róla, ne lődd még le! Köszönöm.

Előzmény: [1136] BohnerGéza, 2008-10-16 19:03:53
[1136] BohnerGéza2008-10-16 19:03:53

Ábra a 138. feladathoz.

Előzmény: [1125] BohnerGéza, 2008-09-24 11:16:44
[1135] BohnerGéza2008-10-12 01:28:32

140/b feladat: Bizonyítandó, ha a 140. feladatban az ABCD négyszög konkáv, akkor az állítás - ha vannak - a közös belső érintőkkel, igaz! ( Szóval a BG és DE átmegy a körök belső hasonlósági pontján. )

Megjegyzés: Ha a 140-ben a két kör sugara egyforma, akkor BG és DE párhuzamos a centrálisukkal.

Előzmény: [1134] BohnerGéza, 2008-10-12 01:13:22
[1134] BohnerGéza2008-10-12 01:13:22

140.feladat: A konvex ABCD négyszögben DA+AB=BC+CD-vel. (Nem érintőnégyszög!) Az ABC háromszög beírt köre érintse AC-t az E, a BDA-é a G pontban. Bizonyítandó, hogy a beírt körök közös külső érintői, a BG és a DE egyenesek egy ponton mennek át!

[1133] sakkmath2008-10-05 16:13:09

Kérdéseddel dr. Dályay Pál Péterhez, a szegedi Deák Ferenc Gimnázium tanárához célszerű fordulni. A feladatot az ő javaslatára a The American Mathematical Monthly tűzte ki 2006-ban 11240-es sorszámmal. A 2008. évi februári szám internetes hivatkozása itt már a rövidített megoldást is közli.

Előzmény: [1132] Lóczi Lajos, 2008-10-03 21:21:38
[1132] Lóczi Lajos2008-10-03 21:21:38

Az exponenciális függvény konvexitása miatt minden valós x-re 1+x\leex. Speciálisan, ha x\in(0,1), akkor 1-x\lee-x, így \frac{1}{1-x}\ge e^x. Legyen most y=\frac{1}{1-x}\in (0,\infty) tetszőleges, ekkor y\ge e^{1-\frac{1}{y}}. Végezzük itt el az y=\frac{(1+\lambda)^2}{4\lambda} helyettesítést, ha \lambda>0 tetszőleges. Azt kapjuk, hogy \frac{(1+\lambda)^2}{4\lambda}\ge e^{\frac{(\lambda-1)^2}{(\lambda+1)^2}}. Legyen most \lambda=\frac{A}{B}, ha A>0 és B>0 tetszőleges. Ekkor \frac{(A+B)^2}{4A B}\ge e^{\frac{(A-B)^2}{(A+B)^2}}. Ha C>0 tetszőleges, akkor betűcserével ebből azt kapjuk, hogy \frac{(A+C)^2}{4A C}\ge e^{\frac{(A-C)^2}{(A+C)^2}} és \frac{(B+C)^2}{4B C}\ge e^{\frac{(B-C)^2}{(B+C)^2}} is igaz. Az utolsó három egyenlőtlenséget összeszorozva azt nyerjük, hogy minden pozitív A, B és C-re


\left(\frac{(A+B)(A+C)(B+C)}{8A B C}\right)^2\ge e^{\frac{(A-B)^2}{(A+B)^2}+\frac{(A-C)^2}{(A+C)^2}+\frac{(B-C)^2}{(B+C)^2}}.

Vonjunk gyököt az egyenlőtlenségből, majd végezzük el az A=a+b-c>0, B=a-b+c>0, C=-a+b+c>0 helyettesítéseket, ahol a, b és c a háromszög oldalai. Azt kapjuk, hogy


\frac{a b c}{(a+b-c)(a-b+c)(-a+b+c)}\ge e^{\frac{(a-b)^2}{2c^2}+\frac{(b-c)^2}{2a^2}+\frac{(c-a)^2}{2b^2}}.

A fenti bal oldal viszont éppen \frac{R}{2r}.

Amúgy honnan jött ez az állítás? (Valami hiperbolikus/abszolút geometriai tétel?)

Előzmény: [1131] sakkmath, 2008-10-03 13:36:48
[1131] sakkmath2008-10-03 13:36:48

139. feladat:

[1130] Káli gúla2008-10-01 16:36:48

Ha K a a háromszög symmediáns pontja és O a köré írt kör középpontja, akkor a Tucker körök középpontjai éppen az OK szakasz pontjai. A bizonyítás megtalálható pl. W. Mclelland: A Treatise On The Geometry Of The Circle. Macmillan, 1891. könyvében a 73. oldalon. A könyv olvasható, és le is tölthető a www.archive.org oldalon, csak rá kell keresni a címére.

Előzmény: [1128] klerox, 2008-09-30 17:45:13
[1129] BohnerGéza2008-09-30 21:46:44

Azt hiszem - így ábra nélkül -, hogy az A'A"=B'B"=C'C" az nem feltétel, hanem igazolható, bárhova vesszük is fel a kezdő A' pontot.

Előzmény: [1128] klerox, 2008-09-30 17:45:13
[1128] klerox2008-09-30 17:45:13

A Tucker körnek a lényege: Feltétel: A'A"=B'B"=C'C", és A'A", B'B" és a C'C" antiparallel a BC, CA és AB oldalakkal. A hat pont: A', A", B', B", C' és C" egy körön helyezkedik el. Tudomásom szerint kell középpontjának lenni.

[1127] BohnerGéza2008-09-29 22:40:25

Biztos, hogy nem a "Matematikus-fizikus viccek, sztorik" közé akartad írni?

Utána néztem, a Tucker-kör vszleg hatszög, azaz önmagába visszatérő vonal. Ilyet a talpponti háromszög (piros) segítségével is meg lehet szerkeszteni. Az ábrán kettő látszik (kék, zöld) és nyilván végtelensok van. Mivel ezen hatszögek köré írható kör, de ezek kp-jai nem esnek egybe, nincs értelme a kérdésnek.

Előzmény: [1126] klerox, 2008-09-29 19:30:34
[1126] klerox2008-09-29 19:30:34

Sziasztok!

A Tucker-kör megszerkesztésével bajlódok, de sehogy nem jövök rá a középpont megszerkesztésére, kérlek segítsetek.

Előre is köszönöm

[1125] BohnerGéza2008-09-24 11:16:44

Ismert a következő: Ha négy egyenes négy háromszöget határoz meg, akkor ezek körülírt körei átmennek egy közös ponton.

138. feladat: Bizonyítandó, hogy a négy körülírt kör középpontja és ez a közös pont egy körön van.

[1123] farkasb2008-09-23 11:18:35

Köszönöm!

Közben sikerült megoldanom más módszerrel a kör és sík problémáját is.

Előzmény: [1122] Sirpi, 2008-09-08 13:41:42
[1122] Sirpi2008-09-08 13:41:42

Nemrég nekem is szükségem volt hasonló programra, csak éppen gömbközéppontot kellett keresni felületi pontok alapján, és írtam egy egyszerű módszert, ami a tesztek alapján elég jól működik, röviden le is írom, hogyan (körre is jó változtatás nélkül):

Legyenek adva a P1,P2,...,Pn pontjaink, és keressük az O középpontot iterációs módszerrel. Legyen O0 a megadott Pi ponthalmaz súlypontja. Ha egyenletes a ponteloszlás, akkor ez már önmagában is majdnem jó, de ha nem, akkor is egy jó kiindulópont.

Nézzük a k+1. lépést: rendezzük sorba a Pi pontokat az Ok-tól vett távolság alapján, és a robusztusság miatt ebből hagyjuk el a legközelebbi és a legtávolabbi 10%-ot, legyen ez első meghagyott pont R (tehát ez egy majdnem legközelebbi pont Ok-hoz képest), az utolsó S (ez pedig a majdnem legtávolabbi, leszámítva a felső 10%-ot). Ideális esetben Ok-t az RS szakasz felezőmerőlegesére kellene "ráhúzni", de tapasztalataim alapján így lassabb a konvergencia, mintha csak a felezőmerőlegeshez képest a felére csökkentenénk a távolságot. Tehát ha Ok talppontja RS felezőmerőlegesére T, akkor Ok+1-nek válasszuk Ok és T felezőpontját, majd folytassuk az eljárást.

Megállási feltétel: Egy adott Ok pont jóságát mérjük az |SOk|-|ROk| különbséggel, és ha ez kisebb, mint az eddigi legjobb, akkor jegyezzük meg ezt, mint lehetséges végső kör/gömbközéppontot. És ha mondjuk 1000 iterációs lépésen keresztül ez a rekord nem dől meg, akkor legyen ez a végső győztes is egyben.

Előzmény: [1121] farkasb, 2008-09-08 00:49:03
[1121] farkasb2008-09-08 00:49:03

Kedves "Fálesz" Mihály!

Visszatérve a legjobban illeszkedő körhöz... Sajnos nem tudom végigszámolni a kör a megadott segédletek alapján, elakadok vele..Az a baj,hogy nem teljesen látom át. Örülnék ha egy konkrét példán bemutatott feladatmegoldást láthatnék.

Van egy programom, ami képes erre a legjobban illeszkedő kör készítésére, a megadott pontokra az alábbi eredmények számolja:

Adottak az alábbi pontok (X,Y):

(0,1000); (10,1000); (400,4000); (500,2000); (-500,-1500); (-900,-3500); (-7000,-3500);

Eredmények(origó,sugár):

X0= -4853.3 ; Y0= 1209.7 ; R= 5352.6 ;

Távolságok az origótól:

1=4857.8; 2=4867.8; 3=5948.4; 4=5411.3; 5=5127.7; 6=6149.0; 7=5175.8;

Amennyiben nem túl nagy munka a segítség, megköszönném. Ha nagy, akkor sincs semmi gond :) Előre is köszönettel: farkasb

Előzmény: [1111] Fálesz Mihály, 2008-09-02 21:45:36
[1120] Sirpi2008-09-07 11:22:30

Mivel semmi más adat nincs megadva a két egyenesen kívül, ezért nem nagyon jutott eszembe más lehetöség, hogy mi lehet a feladat. Felsoroltam azt a 2-t, ami viszont igen :-)

Előzmény: [1118] jonas, 2008-09-03 22:25:25
[1119] BohnerGéza2008-09-03 22:56:06
Előzmény: [1116] Lowosan, 2008-09-03 19:41:43
[1118] jonas2008-09-03 22:25:25

Ezt meg hogy tippelted meg?

Előzmény: [1114] Sirpi, 2008-09-03 15:54:09
[1117] jonas2008-09-03 22:25:02

Azt kell tudni hozzá, hogy számolhatod ki egy pontnak egy egyenestől mért távolságát. Ezután csak veszel egy pontot az egyik egyenesről, és kiszámolod ennek a távolságát a másiktól.

Előzmény: [1116] Lowosan, 2008-09-03 19:41:43
[1116] Lowosan2008-09-03 19:41:43

távolságuk kell elfelejtettem odairni

[1115] Fálesz Mihály2008-09-03 18:41:12

Szerintem először olvasd el újra, és próbáld megérteni, hogy hogyan működik.

Ami írtam, abból kijön az egyenes, a gömb és a sík illesztése is.

Előzmény: [1112] farkasb, 2008-09-03 13:16:31
[1114] Sirpi2008-09-03 15:54:09

Mondjuk segítene, ha elárulnád, mit kell ezzel a két (egyébként párhuzamos) egyenessel csinálni :-) A távolságuk kell, vagy fel kell őket rajzolnod?

Előzmény: [1113] Lowosan, 2008-09-03 15:30:33
[1113] Lowosan2008-09-03 15:30:33

Helló Segitségre lenne szükségem a kordinátageometria területén e:3x+2y=12 f:6x+4y=-12 ezt hogy kéne megoldani? mert új matektanárunk van és nem magyaráz semmit csak annyit mond csináljunk meg semmi más

[1112] farkasb2008-09-03 13:16:31

Kedves Mihály!

Köszönöm a gyors választ, megpróbálkozok a feladattal. Esetleg nem tudna ajánlani valami jó megoldást legjobban illeszkedő sík elkészítésére? A hibák az x y és z koordinátákat is terheljék. Köszönöm!

Előzmény: [1111] Fálesz Mihály, 2008-09-02 21:45:36
[1111] Fálesz Mihály2008-09-02 21:45:36

Vaughan Pratt munkája nyomán a következőt tudom ajánlani. (Én magam is sokszor haszáltam ezt a módszert kör, gömb és henger illesztésére.)

Keressük a kör egyenletét

f(x,y)=A(x2+y2)+Bx+Cy+D=0(1)

alakban, ahol

B2+C2-4AD=1.(2)

Egy kis számolás után látható, hogy a (2) feltétel azzal ekvivalens, hogy a kör sugara R=\frac{1}{2A}, illetve az f(x,y) függvény gradiense a körvonalon egységnyi hosszú.

Ha (x,y) egy pont a körvonalhoz közel, akkor |f(x,y)| jól közelíti a pont és a körvonal távolságát.

Legyenek a mintapontok (x1,y1),...,(xn,yn). Az illesztés négyzetes hibája közelítőleg


\sum_{i=1}^n f^2(x_i,y_i) =
(A,B,C,D) \left(\matrix{ 
\sum(x_i^2+y_i^2)^2 & \sum(x_i^2+y_i^2)x_i 
 & \sum(x_i^2+y_i^2)y_i  & \sum(x_i^2+y_i^2) \cr
\sum(x_i^2+y_i^2)x_i & \sum x_i^2 & \sum x_iy_i & \sum x_i \cr
\sum(x_i^2+y_i^2)y_i & \sum x_iy_i & \sum y_i^2 & \sum y_i \cr
\sum(x_i^2+y_i^2) & \sum x_i & \sum y_i & n \cr }
\right) \left( \matrix{A\cr B\cr C\cr D\cr}\right).
(3)

Szóval a (3) kifejezés minimumát keressük a (2) feltétel mellett. Ha ráereszted a Lagrange-multiplikátor módszert, kapsz egy 4-dimenziós sajátértékfeladatot, ráadásul a keresett sajátérték pont a hiba konstansszorosa...

Ha a (2) helyett az A=1 feltételt használod, akkor a számolás sokkal egyszerűbb, mert csak lineáris egyenletszert kell megoldani. Ennek a gyengéje, hogy a (3)-ban a hibát megszoroztad kb. 4R2-tel. Zajos adat esetén nagyobb lesz az illesztés valódi hibája, de kisebb lesz a sugár, amivel megszorzod.

Előzmény: [1110] farkasb, 2008-09-02 09:00:28
[1110] farkasb2008-09-02 09:00:28

Kedves Fórumozók!

Ismételten egy kérdést tennék fel:

Hogyan határozható meg egy olyan kör középpontja, ami n számú xy koordinátával adott pontra legjobban illeszkedik? Előre is köszönettel: farkasb

[1109] BohnerGéza2008-09-02 01:25:18

Fálesz Mihály a [1105]-ban fölvetette, hogy egy számolás nélküli megoldás szép lenne. Egyelőre nincs hozzá ötletem. A 137/b és 137/c feladat tulajdonképpen megoldása a 137-nek, igaz végig kell gondolni. (Számolással egyszerűbben is megy [1106].)

Előzmény: [1108] BohnerGéza, 2008-08-28 23:51:38
[1108] BohnerGéza2008-08-28 23:51:38
Előzmény: [1107] HoA, 2008-08-28 17:10:45
[1107] HoA2008-08-28 17:10:45

Még tart a nyári szünet... 137/b feladat megoldása: Legyen az F1F2 távolság 2f , F1F2 felezőpontja O. Az F1 középpontú r1 sugarú k1 és az F2 középpontú r2<r1 sugarú k2 körök h hatványvonala messe F1F2-t H -ban. Határozzuk meg az OH távolságot f,r1ésr2 ismeretében. Legyen k1ésk2 közös érintője t , az érintési pontok T1ésT2 , a Ti -ből F1F2 -re bocsátott merőleges talppontja Mi , az F2-n át t-vel húzott párhuzamos és F1T1 metszéspontja L, t és h metszéspontja N. F1L=r1-r2 Az F1M1T1,F2M2T2ésF1LF2 derékszögű háromszögek hasonlóak, F_1 M_1 = \frac{r_1 ( r_1 - r_2)}{2f},  F_2 M_2 = \frac{r_2 ( r_1 - r_2)}{2f} . M_1 M_2 = F_1 F_2 + F_2 M_2 - F_1 M_1 = 2f + \frac{(r_2 - r_1) ( r_1 - r_2)}{2f} .

N felezi T1T2 -t ( egyenlő érintőszakaszok ) , ezért H is felezi M1M2-t. M_1 H = 1/2 M_1 M_2 = f + \frac{(r_2 - r_1) ( r_1 - r_2)}{4f} O H = F_1 M_1 + M_1 H - F_1 O = \frac{r_1 ( r_1 - r_2)}{2f} + f + \frac{(r_2 - r_1) ( r_1 - r_2)}{4f} - f = {\bf \frac{(r_1 + r_2 )( r_1 - r_2)}{4f}}. Feladatunkban r_1 = ( f + d ) \sqrt 2 , r_2 = ( f - d ) \sqrt 2 , r_1 + r_2 = 2 \sqrt 2 f ,  r_1 - r_2 = 2 \sqrt 2 d , igy O H = \frac {2 \sqrt 2 f \cdot 2 \sqrt 2 d }{4f} = { \bf 2d}.

Előzmény: [1106] BohnerGéza, 2008-08-24 23:37:33
[1106] BohnerGéza2008-08-24 23:37:33

Tetszik az affinításos és a többi megoldás is.

Így a 137. feladat általánosításával - miszerint a középvonal 1:1 aránya helyett más arányban osztó szakaszt adunk meg a magasságpont számára - kapott feladat is könnyen bizonyítható.

Érdemes persze Fálesz Mihály ajánlata alapján, igaz számolással, az ellipszis definíciója alapján is elvégezni a bizonyítást!

137/b. feladat: Az A, F1, B, F2 pontok egy négyzet csücsai. Az AC átlóval párhuzamos - tőle d (<>0) távolságra húzott egyenes messe az AF1-t P, az AF2-t Q-ban. Legyen C az F1 kp-ú P-n és az F2 kp-ú Q-n átmenő kör kp-ja. Mutassuk meg, hogy C 2d távolságra van AC-től! (Milyen d esetén vam C pont?)

Előzmény: [1105] Fálesz Mihály, 2008-08-18 10:54:25
[1105] Fálesz Mihály2008-08-18 10:54:25

Legyen M a magasságpont, és T a C-ből induló magasság talppontja. Mint jól ismert, TC . TM = - TA . TB. (Ez azért igaz, mert a magasságpontnak az AB oldalra vonatkozó tükörképe a körülírt körön van, de például az ATC és MTB háromszögek hasonlóságából is láthatjuk.)

Ha az M pont a középvonalon van, akkor TC és TM azonos irányú, TA és TB pedig ellentétes írányú, vagyis T az AB szakasz belsejében van. A TC félegyenes tehát metszi az AB átmérőjű kört; legyen K a metszéspontjuk.

Fálesz Mihály tétele szerint BKA\angle=90o, a magasságtételből pedig TK2=AT . TB.

Így tehát TC2=2.TC.TM=2.TA.TB=2.TK2 és

 TC = \sqrt2\cdot TK,

a C csűcs azon az ellipszisen van, amit az AB átmérőjű körből kapunk az AB tengelyű, \sqrt2 arányú merőleges affinitással.

Az A és B pont kivételével az ellipszis bármelyik pontjából kiindulva a gondolatmenet megfordítható.

* * *

Eleminek ezt a megoldást sem nevezném, bár az eredmény alapján nehéz ennél "elemibbet" várni. Aki akarja, megpróbálhatja -- számolás nélkül!!! -- bebizonyítani, hogy F1C+F2C=F1A+F2A, csak ezzel meg az a bajom, hogy az eredményt valahonnan máshonnan tudtuk meg.

Előzmény: [1104] HoA, 2008-08-15 15:20:47
[1104] HoA2008-08-15 15:20:47

Ne feledkezzünk meg a 137. feladatról. Kedvcsinálónak BohnerGéza sejtésének igazolása. Ha a magasságpont az FaFb középvonalon van, akkor a körülírt kör K középpontja a középponti háromszög középvonalán van, tehát K távolsága a c oldaltól negyede mcnek. Az általánosság megszorítása nélkül legyenek A, B és C koordinátái (-1;0) , (1;0) és (x,y) . Ekkor K =(0;y/4) . KB = KC -ből a távolságok négyzetére

1+(y/4)2=x2+(3y/4)2

. 1+y2/16=x2+9y2/16 ; 1=x2+8y2/16

\frac{x^2}{1}+\frac{y^2}{2} = 1

A C pontok egy ellipszisen helyezkednek el. Várjuk az elemi geometriai megoldásokat!

Előzmény: [1085] BohnerGéza, 2008-07-20 21:50:08
[1103] BohnerGéza2008-08-12 21:24:34

Köszönöm HoA, köszönöm Fálesz Mihály!

Örülök, hogy ilyen szép megoldások készültek itt erre az olimpiai feladatra.

Előzmény: [1102] HoA, 2008-08-12 16:42:59
[1102] HoA2008-08-12 16:42:59

Köszönöm, kitűnő gondolat. Pedig már az ábrán is látszott... Mivel ka és kb egyik metszéspontja H, hatványvonaluk a H-n áthaladó, FAFB-re merőleges egyenes, vagyis mc. C rajta van a hatványvonalon, CA1.CA2=CB1.CB2, vagyis A1,A2,B1,B2 egy körön van.

Előzmény: [1100] Fálesz Mihály, 2008-08-12 09:22:15
[1101] BohnerGéza2008-08-12 12:30:42
Előzmény: [1098] Káli gúla, 2008-08-11 21:23:53
[1100] Fálesz Mihály2008-08-12 09:22:15

Tessék számolás nélkül megoldani.

Segítség: Az A1A2 és B1B2 körök egyik metszéspontja a H magasságpont. Hol lehet a másik metszéspont?
Előzmény: [1099] HoA, 2008-08-05 11:27:07
[1098] Káli gúla2008-08-11 21:23:53

A 9-es szorzót trigonometria nélkül is megkaphatjuk: Ha a,b,c három egységvektor, azaz |a|=|b|=|c|=1, akkor

(a-b)2+(a-c)2+(b-c)2=2(a2+b2+c2)-2ab-2ac-2bc=3(a2+b2+c2)-(a+b+c)2=9-(a+b+c)2\le9 .

Egyenlőség pontosan akkor van, ha a+b+c=0. Ugyanígy, n darab egységvektor közötti összes távolság négyzetösszege legfeljebb n2 (és egyenlőség csak \sum x_i=0 esetén lehet):

\sum_{i<j} (x_i-x_j)^2 =
 \sum_{i<j} (x_i^2 + x_j^2) - \sum_{i<j} 2 x_i x_j 
 = (n-1) \sum x_i^2 - \left[(\sum x_i)^2 - \sum x_i^2\right]=
  n^2 - (\sum x_i)^2 \le n^2

Előzmény: [1093] BohnerGéza, 2008-08-05 20:34:02
[1097] BohnerGéza2008-08-10 21:33:21
Előzmény: [1096] HoA, 2008-08-08 12:06:40
[1096] HoA2008-08-08 12:06:40

[1096] - hoz hasonlóan a cosinus tételből kiindulva kicsit körülményesebben is erre az eredményre juthatunk :-) . Az adott körbe írt H0 háromszögben, mint [1096] ábráján is , legyen \gamma a nagyságra középső szög, \alpha\ge\gamma\ge\beta . Ekkor 0 < \gamma < \frac{\pi}{2} . Rögzítsük c-t , és így \gamma-t is . a2+b2+c2=2.c2+2abcos\gamma . Ez cos\gamma>0 miatt annál nagyobb, minél nagyobb az ab szorzat. A sinus tételből ab = \frac{c^2 sin \alpha sin \beta}{sin^2 \gamma} , tehát elég sin\alphasin\beta változását vizsgálni. A szorzatot összeggé alakítva sin\alphasin\beta=1/2{cos(\alpha-\beta)-cos(\alpha+\beta)} . Állandó \gamma mellett \alpha+\beta is állandó, a két szög különbségének cosinusa pedig annál nagyobb, minél kisebb a két szög különbsége. Eljutottunk [1096] eredményéhez: C -t a körülírt körön az AB ív F felezőpontja felé mozgatva \alpha-\beta csökken és így az oldalak négyzetösszege nő.

Mindkét megközelítésre vonatkozó megjegyzés: Szigorúan véve csak azt mutattuk meg, hogy ha van az oldalak négyzetösszegének maximuma, az csak a szabályos hármszögben lehet. Kis finomítással bizonyítható a maximum létezése. H0-ban \alpha>60o és \beta<60o , mint a háromszög legnagyobb ill. legkisebb szöge. Ha C -vel F -be jutunk, a másik két szög egyenlő lesz, tehát áthaladtunk egy olyan C' pozíción, ahol a két szög egyike éppen 60o . Legyen ez a háromszög H1 . H1 -ben tehát az oldalak négyzetösszege nagyobb, mint H0-ban. Betűzzük át H1 csúcsait úgy, hogy itt is teljesüljön \alpha\ge\gamma\ge\beta . Nyilván \gamma lesz a 60o -os szög. H1 -ben c-t rögzítve és C -vel a körülírt körön mozogva az oldalak négyzetösszege az AB ív F felezőpontjában nagyobb lesz, mint H1 -ben . Ez a H2 háromszög viszont szabályos, hiszen \gamma=60o és \alpha=\beta. Tehát tetszőleges H0-ból indulva a szabályos H2 -ben nagyobb az oldalak négyzetösszege , mint H0-ban , H2 tényleg a maximumot adja.

Második megjegyzés: [1096] helyes ábrájához a szövegben is meg kell említeni, hogy \gamma-nak az egyik hegyesszöget választjuk. Ellenkező esetben nagyobb területhez kisebb (tompaszög) vagy változatlan (derékszög) oldal-négyzetösszeg tartozik.

Előzmény: [1095] BohnerGéza, 2008-08-08 01:43:50
[1095] BohnerGéza2008-08-08 01:43:50

Következnek-e az alábbi levezetésből az előző hozzászólásban HoA által kinondtt állítások?

Előzmény: [1094] HoA, 2008-08-07 13:11:34
[1094] HoA2008-08-07 13:11:34

Mivel szabályos háromszögre a2+b2+c2=9R2, és próbálgatással más háromszögekre a 9-et sem sikerült elérni, feltételeztem, hogy a2+b2+c2\le9R2 is igaz. Az oldalakat R-rel és a háromszög szögeivel kifejezve ebből sin2\alpha+sin2\beta+sin2\gamma\le \frac94 adódik. Biztos volt már KöMaL vagy versenyfeladat, én most itt találtam meg, mint a "világ hét csodájának" egyikét: http://mathcircle.berkeley.edu/trig.pdf ( Bizonyítással együtt, aki próbálkozni akar, csak utána nézze meg )

Előzmény: [1093] BohnerGéza, 2008-08-05 20:34:02
[1093] BohnerGéza2008-08-05 20:34:02

((Az alábbiakból kiderül, hogy a surár négyzetének tízszerese nagyobb az oldalak négyzetösszegénél. Van jobb összefüggés?))

Előzmény: [1099] HoA, 2008-08-05 11:27:07
[1099] HoA2008-08-05 11:27:07

Hát ha senkit nem érdekel... A [1087] feladat megoldása: Mivel A1A2 a keresett kör húrja, ennek felező merőlegese egyben a BC oldal felező merőlegese, tehát átmegy K-n, és ez a másik két oldalra is igaz, így ha van ilyen kör, annak középpontja csak K lehet. A javasolt segítséget felhasználva - legyen a helyvektorok kezdőpontja K - írjuk fel K és A2 távolságát, felhasználva, hogy ekkor \vec{H} = \vec{A} + \vec{B} + \vec{C} . Legyen BC felezőpontja  \vec{F_A} = \frac{\vec{B} + \vec{C}}2 , \vec{H} - \vec{F_A} = \vec{A} +\vec{B} + \vec{C} - \frac{\vec{B} + \vec{C}}2 = \frac{2\vec{A} + \vec{B} + \vec{C}}2 . Ennek hossza FA és A2 távolsága , ezért

{\vec{A_2}}^2 = (\vec{H} - \vec{F_A})^2 + {\vec{F_A}}^2  = \left(\frac{2\vec{A} + \vec{B} + \vec{C}}2\right)^2 + \left (\frac{\vec{B} + \vec{C}}2\right)^2 =\frac{4\vec{A}^2 + 2(\vec{B} + \vec{C})^2 +4\vec{A}(\vec{B} + \vec{C})}4 =

= \frac{4\vec{A}^2 + 2\vec{B}^2 + 2\vec{C}^2 + 4\vec{B}\vec{C}+4\vec{A}\vec{B} +4\vec{A} \vec{C}}4 = 2\vec{R}^2 + \vec{B}\vec{C}+\vec{A}\vec{B} +\vec{A} \vec{C}.

Ez a kifejezés A, B, C -ben szimmetrikus, tehát például K és B2 távolságának négyzetére ugyanezt kapnánk, a hat pont így egy K középpontú körön fekszik.

Hasonló, de trigonometriai átalakításokat igénylő megoldást kapunk, ha a feltételezett Rh körsugarat a körülírt kör R sugarával és a \Delta szögeivel fejezzük ki. Legyen BC felezőpontja FA, ma talppontja TA . Rh2=KFA2+FAA22=KFA2+FAH2=KFA2+FATA2+TAH2 .

Itt KFA=R.cos\alpha ,

BTA=2.R.sin\gammacos\beta , FATA=2.R.sin\gammacos\beta-R.sin\alpha ,

TAH=2.R.sin\gammacos\beta.ctg\gamma=2.R.cos\beta.cos\gamma . A négyzetösszeg:

Rh2=R2(cos2\alpha+4.cos2\beta.sin2\gamma+sin2\alpha-4.sin\alphacos\betasin\gamma+4.cos2\betacos2\gamma)=R2(1+4.cos2\beta-4.sin\alphacos\betasin\gamma)=R2(1+4.cos\beta(cos\beta-sin\alphasin\gamma))=R2(1+4.cos\beta(-cos(\alpha+\gamma)-sin\alphasin\gamma))=R2(1+4.cos\beta(sin\alphasin\gamma-cos\alphacos\gamma-sin\alphasin\gamma))=R2(1-4.cos\alphacos\betacos\gamma)

Mivel a kifejezés \alpha,\beta,\gamma -ban szimmetrikus, az első megoldáshoz hasonlóan adódik, hogy a 6 pont valóban egy K középpontú Rh sugarú körön fekszik.

Előzmény: [1087] BohnerGéza, 2008-07-22 01:24:34
[1092] HoA2008-07-23 13:35:46

A 136. feladat megoldása: Induljunk ki a 120o -nál nem nagyobb szögű \Delta esetére ismert bizonyítás ábrájából. Forgassuk el a CAP \Delta -et az óramutató járásának irányába C körül 60o -kal, a másik két csúcs elforgatottja A' ill. P' . AA'C és PP'C \Delta -ek szabályosak, AA' = AC és PP' = PC, valamint A'P' = AP , tehát azt kell igazolnunk, hogy a BPP'A' T2 törtvonal nem rövidebb a BAA' T1 törtvonalnál. Az összehasonlításnál a nehézséget csak az jelenti, hogy a két törtvonal - mint ábránkon is - keresztezheti egymást. Egészítsük ki ezért az A'P'P \Delta -et a PA' átlójú A'P'PP'' parallelogrammává. Ekkor PP'' = P'A' és P''A' = PP', így T2 helyett vehetjük a vele azonos hosszúságú BPP''A' T3 törtvonalat. A'P' AP 60o-os elforgatottja, így a vele egyállású P''P PA-val 60o-os szöget zár be, APP'' \Delta szabályos, AP'' AP 60o-os órajárással ellentétes elforgatottja. Mivel AP az ABC \Delta belsejében, és így a konvex BAC szögtartomány belsejében ( vagy határán ) fekszik, AP'' a BAA' konkáv szögtartomány belsejében ( vagy határán ) fekszik, ezért az APP'' \Delta és így T3, a BPP''A' törtvonal is a a BAA' konkáv szögtartományba esik, P vagy P'' esetleg a határára. Legyen az AA' egyenes és T3 metszéspontja D. Jelöljük X és Y pontok T3 mentén mért távolságát tXY-nal. Ekkor tBD+DA\geBA és tDA'\geDA'=DA+AA' . A kettőt összeadva DA kiesik: tBD+tDA'=tBA'\geBA+AA', és egyenlőség csak akkor áll fenn, ha mindkét egyenlőtlenségben fennáll, vagyis ha P = P'' = A.

Előzmény: [1078] BohnerGéza, 2008-07-15 17:42:46
[1091] farkasb2008-07-23 04:59:18

Kedves BohnerGéza!

Köszönöm a helyreigazítást. Én normálvektornak már egy egységvektort írtam

Előzmény: [1090] BohnerGéza, 2008-07-22 21:37:40
[1090] BohnerGéza2008-07-22 21:37:40

Kedves farkasb!

Nekem (-17;12;1) jött ki normálvektornak.

Természetesen örülök, hogy matekkal foglalkozol. Viszont "a normálvektor" kifejezés nem jó! Egy normálvektort nullától különböző számmal szorozva szintén azt kapunk.

( normálvektorod amúgy jó )

Előzmény: [1088] farkasb, 2008-07-22 18:39:20
[1089] farkasb2008-07-22 19:53:38

Közben sikerült rájönnöm...

Előzmény: [1088] farkasb, 2008-07-22 18:39:20
[1088] farkasb2008-07-22 18:39:20

Kedves Fórumozók!

Lenne egy elvileg egyszerű kérdésem, nekem nem megy...nem találtam rá sehol megoldást, pedig tényleg egy alapfeladat. Adott egy sík 3 pontjával: A(1,-1,2) B(4,3,5) C(3,2,0)

Melyből a normálvektor:

n(-0.816,0.576,0.048)

És egy P(8,5,1) pont

Keresett a P pont síkon lévő merőleges vetületének xyz koordinátája.

Előre is köszönettel a segítségért: Balázs

[1087] BohnerGéza2008-07-22 01:24:34
[1086] BohnerGéza2008-07-21 01:02:26

Nem voltam biztos benne, most ellenőriztem, az [1083]-ban lévő négyszög húrnégyszög is.

Előzmény: [1084] m2mm, 2008-07-19 11:02:32
[1085] BohnerGéza2008-07-20 21:50:08

( Olyan jellegű, mint érintő négyszög esetén.)

Az alábbi feladatban, azt hiszem, másodrendű görbe a megoldás.

137. feladat: Adott az A és B pont. Határozd meg azon C pontok halmazát, melyekre az ABC háromszög magasságpontja az FaFb középvonalon van.

Előzmény: [1084] m2mm, 2008-07-19 11:02:32
[1084] m2mm2008-07-19 11:02:32

Köszönöm a választ, de mi P pont mértani helye akkor, ha nem érintőnégyszögről van szó, hanem húrnégyszögről(ennél találtam én is jó pontokat, például a köréírt kör középpontja, és az AB és CD egyenesek metszéspontja)?

Előzmény: [1083] BohnerGéza, 2008-07-19 02:14:25
[1083] BohnerGéza2008-07-19 02:14:25

Legyen q= PA/PD = PC/PB. Ez azt jelenti, hogy az AD és a CB szakasz q arányú Appolonius-köre a P pontban metszik egymást.

Euklides-ben felvéve egy változtatható arányt (egy állandó szakaszt és "futópont" segítségével egy változó hosszút), megszerkesztve a két szakasz ilyen arányú Appolonius-körét, ezek metszéspontjának nyomvonala kirajzolható. (Sajnos az adatokkal játszani kell, hogy valóban elég hosszú részletét láthassuk.)

Játszadozásom azt valószínűsíti, hogy általában nem másodrendű görbe vonala a keresett halmaz.

Előzmény: [1079] m2mm, 2008-07-15 19:12:47
[1082] m2mm2008-07-16 17:53:31

Nem vektorok szorzata, hanem a PA és PB szakaszok szorzata valamint a PC és PD szakaszok szorzata.

Előzmény: [1081] HoA, 2008-07-16 13:28:18
[1081] HoA2008-07-16 13:28:18

És ha igen, akkor skalár vagy vektor szorzat? A vektor szorzat nem valószínű, mert akkor a feltétel egyenértékű azzal, hogy a PAB és PCD háromszőgek (előjeles?) területe egyenlő. Ekkor a P pontnak az AB egyenestől mért távolsága úgy aránylik a CD egyenestől mért távolsághoz, mint a CD és AB szakaszok hossza egymáshoz, a mértani hely egyenes ( ha csak az abszolút értéket vesszük, egyenespár ) , és nem használtuk fel, hogy ABCD érintőnégyszög.

Előzmény: [1080] BohnerGéza, 2008-07-16 02:23:08
[1080] BohnerGéza2008-07-16 02:23:08

Vektorok szorzata?

Előzmény: [1079] m2mm, 2008-07-15 19:12:47
[1079] m2mm2008-07-15 19:12:47

Van egy feladat, aminek a megoldására még nem sikerült rájönnöm, de érdekelne. A feladat: Adott egy nem trapéz érintőnégyszög, csúcsai A,B,C,D. Mi azon P pontok mértani helye a síkon, melyekre PA.PB=PC.PD?

[1078] BohnerGéza2008-07-15 17:42:46

Ismert, hogy ha egy háromszög minden szöge kisebb 120 foknál, akkor az izogonális pont (melyből a háromszög minden oldala 120 fokos szögben látszik) az a pont, melytől a csúcsokig mért szakaszok összege minimális. Látható pl. a [270]. hozzászólásban.

136. feladat: Legyen az ABC háromszög A-nál lévő szöge legalább 120 fok. Bizonyítandó, hogy a PA+PB+PC összeg akkor minimális, ha P=A!

[1077] HoA2008-06-23 18:07:40

Segédtétel: A szabályos 18-szög A2A10,A5A12ésA9A18 átlói egy közös M ponton mennek át.

Legyen A2A10ésA9A18 metszéspontja M. Ekkor A2A10 A9A18-ra vonatkozó tükörképe, A8A16 is áthalad M-en. A2A10A8\angle=A16A8A10\angle=60o , A8A10M\Delta szabályos, A10M=A10A8 . De A10A8=A10A12 , mint egyenlő középponti szögekhez tartozó húrok, tehát A10M=A10A12 , MA10A12\Delta egyenlőszárú. MA10A12\angle=A2A10A12\angle=80o , A_{10} A_{12}M \angle = \frac{180-80}{2} = 50^{\circ} . De A10A12A5\angleis50o , tehát M rajta van A5A12-n is.

Előzmény: [1076] HoA, 2008-06-23 17:35:18
[1076] HoA2008-06-23 17:35:18

A KöMaL régebbi olvasói számára ismert, hogy az ilyen feladatok megoldásához, ahol a szögek 10o egész számú többszörösei, jól használható a szabályos 18-szög oldalaiból, átlóiból és körülírt K köréből álló H18 hálózat. Ha ívhossz egységnek K két szomszédos csúcs közötti ívét vesszük, n egységnyi ívhez n.10o kerületi és n.20o középponti szög tartozik. ( Ld. [918] )

Legyenek H18 csúcsai A1,.., A18, K középpontja O, sugara R=1. Húzzuk be az ábrán színessel jelölt átlókat. Az A1A10A12 derékszögű \Delta -ben A10A12=2.sin20o . A12A10A16\angle=A2A10A16\angle=40o , A5A12A10\angle=50o . A5A12 metszéspontja A2A10 -zel M, A10A16-tal N. A10NA12 és A10NM egybevágó derékszögű \Delta-ek, A12N=NM=2.sin20o.sin40o , A12M=4.sin20o.sin40o . Mint azt a segédtételben mrgmutatjuk, M rajta van A9A18-on. A12 tükörképe A9A18-ra A6, A6A12 T-ben merőlegesen metszi A9A18-et. MTA12 derékszögű \Delta -ben MA12T\angle=A5A12A6\angle=10o , A12T=A12M.sin80o=4.sin20o.sin40o.sin80o . A12T egyúttal az OA9A12 egységnyi oldalú szabályos \Delta magassága, tehát  A_{12} T = \frac{\sqrt3}{2} A12T két kifejezéséből a feladat állítása adódik.

Előzmény: [1067] S.Ákos, 2008-06-17 20:28:38
[1075] Fálesz Mihály2008-06-18 18:55:49

Val'szeg az általános iskolás könyvekben megtalálod.

De biztos lesz olyan, aki hajlandó lerajzolni.

Előzmény: [1072] kromers, 2008-06-18 14:13:43
[1074] Fálesz Mihály2008-06-18 18:53:28

Sőt, ragaszkodjunk az első, ógörög nyelvű kiadáshoz.

Előzmény: [1073] Róbert Gida, 2008-06-18 16:58:50
[1073] Róbert Gida2008-06-18 16:58:50

Eukleidesz Elemek című könyvében.

Előzmény: [1072] kromers, 2008-06-18 14:13:43
[1072] kromers2008-06-18 14:13:43

Szép napot mindenkinek! Egy kis segitségre lenne szükségem. Adott egyenessel adott ponton át párhuzamos egyenes szerkesztése. A szerkesztés menetének több változata is ismeretes. A problémám a következő, hol találom meg ezeknek a BIZONYITÁSÁT? Előre is köszönöm

[1071] Euler2008-06-18 14:07:27

Nekem is van egy megoldásom, sajnos ehhez is kell trigonometria, de azért érdermes megnézni:(nem tudok képleteket beírni, így egy kicsit hülyén fog kinézni) szorozd be mindkét oldalt 2cos20-al, vedd észre a sin(2x)=2sinxcosx addicios tételt, majd a sin40 négyzetére ismert addicios összefüggést használd( 2sinx.sinx=1-cos(2x). Ezekután ismételten használd a sin160=2sin80cos80 összefüggést, sin160=sin20, kapod, hogy sin80=1/2sin20+(gyök3)/2.cos20, ez pedig nyilván igaz, hiszen a jobb oldalon sin(x+y) áll.

Előzmény: [1067] S.Ákos, 2008-06-17 20:28:38
[1070] S.Ákos2008-06-18 10:52:36

köszönöm szépen a segítséget, valami ilyesmire gondoltam:)

Előzmény: [1069] Káli gúla, 2008-06-17 22:24:03
[1069] Káli gúla2008-06-17 22:24:03

Kevesebb trigonometriával: A szabályos 9-szögben felírjuk a Ptolemaiosz-tételt az A1A2A4A5 négyszögben \matrix{}a_1^2+a_2 a_4 = a_3^2=4\sin^2 60^\circ=3. Ezt a1-gyel szorozva : \matrix{} a_1a_2a_4=3a_1-a_1^3. (Sajnos a végén a sin 3\alpha képlet itt is kell.)

Előzmény: [1067] S.Ákos, 2008-06-17 20:28:38
[1068] Káli gúla2008-06-17 21:09:36

Ha \alpha=20o, akkor : A=sin \alphasin (3\alpha-\alpha)sin (3\alpha+\alpha)=sin \alpha(sin23\alphacos2\alpha-cos23\alphasin23\alpha).

Ide beírva, hogy \sin 3\alpha=\frac{\sqrt3}{2}, \cos 3\alpha=\frac12, majd összevonva éppen az jön ki:

A=\frac34\sin\alpha-\sin^3\alpha=\frac14 \sin 3\alpha=\frac{\sqrt3}{8}.

Előzmény: [1067] S.Ákos, 2008-06-17 20:28:38
[1067] S.Ákos2008-06-17 20:28:38

Valaki tudna mutatni egy elemi geometriai megoldást az alábbi feladatra: Bizbe, hogy \sin20^\circ\sin40^\circ\sin80^\circ=\frac{\sqrt3}{8}?

[1066] bronco2008-06-12 01:34:40

Neked is köszönöm szépen, sokat segítettél!

[1065] Csimby2008-06-11 23:36:54

segítség A pontokat mozgatni lehet. Ha kettőt egymásra teszel, akkor épp 5 érintőt kapsz és abból már látszik hogy kell a szerkesztést csinálni. (veszed az 5 egyenes által meghatározott 5-szöget, kiválasztod az egyik a oldalát, két szomszédos oldal legyen b és c. A b és c re eső 4 csúcsot "keresztbe" összekötöd, a metszéspon legyen M. Ekkor M-en és az a-val szemben lévő csúcson átmenő egyenes kimetszi a-ból az a-ra eső érintési pontot)

Előzmény: [1064] bronco, 2008-06-11 23:21:28
[1064] bronco2008-06-11 23:21:28

Sziasztok!

Egy projektív geometriai feladatban lenne szükségem segítségre:

Adott egy kúpszelet öt érintőegyenese. Szerkeszzük meg az egyiken az érintési pontot!

Egyszerűnek tűnik, de sehol sem találok hasonló megoldott feladatot. (Annyi tudok, hogy a Brianchon-tétel elfajuló esetét kell alkalmazni.)

Előre is köszi!

[1063] Euler2008-06-05 14:08:04

Köszönöm szépen a megoldást! Nagyon szép és valóban elemi megoldás.

Előzmény: [1062] SmallPotato, 2008-06-05 11:53:17
[1062] SmallPotato2008-06-05 11:53:17

A feladat értelmében a paralelogramma egyik átlója adott (a kör sugara), továbbá szögei is adottak.

Rajzoljuk fel az ismert átlóra az ismert szög látókörét. Az átló végpontjai és a látókör-ív egy futó pontja olyan háromszöget határoznak meg, amelynek területe a paralelogramma területének fele. E háromszög területe viszont akkor maximális, ha a rögzített oldalhoz tartozó magassága maximális, azaz a látókör-ív felében álló harmadik csúcs esetén. Eszerint a paralelogramma területe ott maximális, ahol a paralelogramma oldalai egyenlők, azaz ha a paralelogramma rombusz, vagyis ha a hiányzó negyedik csúcs a körcikk ívének felezőjében van.

Előzmény: [1061] Euler, 2008-06-05 11:30:15
[1061] Euler2008-06-05 11:30:15

Hello mindenkinek! Van egy geometria feladatom, melyre teljesen elemi geometriai megoldást szeretnék hallani, már kijött trigonometriával, de léteznie kell elemi geos megoldásnak is. Ha valakinek van ötlete, legyen szives ossza meg velem is! Előre is köszi, a feladat: Adott egy körcikk, melynek középponti szöge hegyesszög. A körcikkbe olyan paralalogrammákat irunk, amelynek oldalai párhuzamosak a szögszárral, egyik csúcsa a szögcsúcs, 2 a szárakon van és a negyedik a köríven. Mikor maximális a területe a paralelogrammának?

[1060] BohnerGéza2008-06-05 10:52:40

Nem! Kellett és szép volt a megoldásod! Ez utóbbi megoldás, csak a 134/a feltételei mellett megy.

Egymást metsző köröket nem lehet koncentrikussá képezni, illetve, ha a két egymást nem metszö körhöz tartozó mindkét nullkörön átmegy a harmadik, akkor annak képe egyenes lesz.

Előzmény: [1059] HoA, 2008-06-05 00:02:58
[1059] HoA2008-06-05 00:02:58

Hát igen. És ezek szerint jól sejtettem, hogy a 134. feladat megoldása is ez: legyen I az az inverzió, amely k1 -et és k2 -t koncentrikus körökbe képezi le, ekkor akárhová is esik k3' középpontja, a három körközéppont valójában kettő, és így egy egyenesen vannk.

Előzmény: [1058] BohnerGéza, 2008-06-04 20:29:36
[1058] BohnerGéza2008-06-04 20:29:36

Talált! Bocs!

A 135. feladat megoldása: A két nullkörös körsorból koncentrikus lesz, ha az egyik nullkör kp-ú inverzióval képezzük le.

Előzmény: [1057] HoA, 2008-06-04 15:52:46
[1057] HoA2008-06-04 15:52:46

Csak nem az a galád trükk lett volna a 134. feladat eredeti megoldása, hogy ha három pontból kettő egybeesik, akkor egy egyenesen vannak? Az eredeti kitűzésbe ez is belefér. A 135. feladat ábrája ismerős a "Fejezetek az elemi geometriából" c. könyvből. Ez az ottani feladat általánosítása. Eredetileg azt kellett igazolni, hogy ha "1" valamilyen helyétől indulva a körsorozat n körből áll és "n" érinti "1" -et, akkor ez "1" tetszőleges helyéről indulva is fennáll. A kapcsolatot 134. -gyel ott vélem felfedezni, hogy 135. megoldásának kulcsa is két egybeeső körközéppont.

Előzmény: [1056] BohnerGéza, 2008-06-04 12:50:33
[1056] BohnerGéza2008-06-04 12:50:33
Előzmény: [1055] HoA, 2008-06-04 10:00:34
[1055] HoA2008-06-04 10:00:34

A 134/a feladat megoldása: Az egymást nem metsző k1 és k2 körök által maghatározott körsor legyen KS1 , nullkörei N1 és N2 . k3 ne haladjon át N2-n, a k3 és N2 által maghatározott körsor legyen KS2 , ennek másik nullköre N3 , a keresett inverzió I, alapköre C, ennek középpontja O. Mivel egy elliptikus körsor köreinek középpontjai a nullkörök által meghatározott egyenesen vannak, köreink inverzeinek, k1',k2'ésk3' köröknek a középpontjai csakkor lesznek egy egyenesen, ha N1' N2' és N3' is. N1' és N2' e1' egyenese I-nél a KS1 körsor ortogonális korsora, KO1 azon ko1i körének lesz az inverze, amelyik O-n áthalad, vagyis az N1 , N2 és O pontok által meghatározott körnek. Hasonlóan N2' és N3' e2' egyenese a KS2 körsor ortogonális korsora, KO2 azon ko2j körének lesz az inverze, amelyik O-n áthalad, vagyis az N2 , N3 és O pontok által meghatározott körnek. e1' és e2' tehát csakkor azonos, ha ko1i és ko2j azonosak, vagyis ko1i = ko2j az N1 , N2 és N3 pontok által meghatározott kör, melyre O is illeszkedik. Ez persze az ortogonalitás miatt megegyezik [1049] e körével, így a megoldás folytatható az ott leírtak szerint, a nullkörök és körsorok megszerkesztése nem szükséges. Az egyszerűbb gyakorlati kivitelezést eredményező O és C megválasztás ebben a terminológiában úgy fogalmazható, hogy legyen C a KO1 azon köre, melynek O középpontja e -re illeszkedik. Ekkor KS1 inverze önmaga.

Előzmény: [1053] BohnerGéza, 2008-05-30 23:09:00
[1054] HoA2008-06-02 13:42:59

További segítségként a 134/a feladathoz feloldom az [1048]-beli észrevételt: Két kör által meghatározott körsor nullköreinek inverze adott inverziónál megegyezik a körsor inverzének nullköreivel.

Előzmény: [1053] BohnerGéza, 2008-05-30 23:09:00
[1053] BohnerGéza2008-05-30 23:09:00

Pontosítom a feltételt, ezzel segítek is: 134/a feladat: A 134. feladatban tegyük meg ezt a kikötést: a 3 kör közt van kettő, melyek nem metszik egymást, és az általuk meghatározott körsor legalább egyik nullkörén nem megy át a harmadik kör. Ekkor HoA előbbi megoldásán túl adódik egy másik megoldás is.

Előzmény: [1052] BohnerGéza, 2008-05-28 20:19:50
[1052] BohnerGéza2008-05-28 20:19:50

Elég, ha van két kör melyeknek nincs közös pontja.

Előzmény: [1051] BohnerGéza, 2008-05-28 20:13:06
[1051] BohnerGéza2008-05-28 20:13:06

Köszönöm HoA megoldását, egyben kitűzöm könnyítve a feladatot:

134/a feladat: A 134. feladatban tegyük meg azt a kikötést, hogy a 3 körnek nem lehet közös pontja. Így az előbbi mo-tól eltérő is megadható!

Előzmény: [1050] HoA, 2008-05-28 08:11:09
[1050] HoA2008-05-28 08:11:09

2. ábra

Előzmény: [1049] HoA, 2008-05-28 08:10:03
[1049] HoA2008-05-28 08:10:03

Legyenek az adott körök k1,k2ésk3, a keresett inverzió I, alapköre C, ennek középpontja O. A körök inverzei k1',k2'ésk3' , e három kör közös szimmetriatengelye e' . Mivel az inverzió saját inverz művelete, k1',k2'ésk3' inverze I-nél k1,k2ésk3. O nem lehet e'-n, mert ekkor a szimmetria miatt k1,k2ésk3 is közös szimmetriatengellyel rendelkezne és nem lehet valamelyik ki'-n sem, mert akkor ki egyenes lenne. Legyen e' inverze e. e-ről megállapítható, hogy

-kör, mert O nincs e'-n

-átmegy O-n, mert e' egyenes

-merőlegesen metszi a k1,k2ésk3 köröket az inverzió szögtartó tulajdonsága miatt.

Adottnak véve k1,k2ésk3 köröket, a keresett inverzióra szükséges feltétel, hogy O középpontja egy olyan e körön legyen, amelyik merőlegesen metszi a k1,k2ésk3 köröket , de ne legyen a körök egyikén sem, hiszen akkor ki' egyenes lenne. Megmutatjuk, hogy a feltétel elégséges is. Legyen e a k1,k2ésk3 köröket merőlegesen metsző kör, O ennek egy - a körökkel nem közös - pontja. Az O középpontú inverziónál a k1,k2ésk3 körök képe három kör lesz, k1',k2'ésk3'. e képe az e' egyenes, amely a szögtartó tulajdonság miatt merőlegesen metszi k1',k2'ésk3' köröket. Mivel egy kört metsző egyenesek közül csak az átmérő egyenesek metszenek merőlegesen, e' mindhárom körnek átmérő egyenese, tehát a három körközéppont egy egyenesen van.

e szerkesztéséhez felhasználjuk, hogy merőlegesen metsző körök esetén az egyik kör metszéspontba mutató sugara a másik kör érintője. e középpontjából, P-ből tehát a k1,k2ésk3 körökhöz egyenlő érintők húzhatók, P a három kör hatványpontja. A szerkesztés menete:

- megszerkesztjük k1,k2ésk3 P hatványpontját mint a körpárok hatványvonalainak metszéspontját - megszerkesztjük a P középpontú, k1-et merőlegesen metsző e kört - e tetszőleges, a körökre nem illeszkedő pontját a kersett inverzió O középpontjául választva tetszőleges sugarú C alapkörrel végrehajtjuk az inverziót. (1. ábra)

Egyszerűbb a gyakorlati kivitelezés, ha O-t úgy választjuk, hogy két kör, például k1ésk3 hatványvonalára illeszkedjék. Ekkor C úgy választható, hogy k1-et és k3-at merőlegesen messe, így ezek az inverzió során önmagukra képeződnek (2. ábra a következő hozzászólásban)

Előzmény: [1042] BohnerGéza, 2008-05-18 23:09:18
[1048] HoA2008-05-21 15:53:40

Segítség a 134. feladathoz:

- Ha három pont nincs egy egyenesen, akkor egy körön vannak

- Melyik három pont? ( A körközéppont inverz képe általában nem az inverz kör középpontja.)

Előzmény: [1042] BohnerGéza, 2008-05-18 23:09:18
[1047] Kata122008-05-21 11:43:02

Kedves Géza!

Nagyon szépen köszönk minden segítséget! Hasznos volt.

[1046] BohnerGéza2008-05-21 00:13:39
Előzmény: [1043] Kata12, 2008-05-20 16:00:04
[1045] Kata122008-05-20 18:45:52

Szia Géza!

Nagyon köszönöm a segítséget, esetleg az háromszöges példát tudnád kicsit részletesebben leveztni ha lehet ábrával?

[1044] BohnerGéza2008-05-20 17:46:55
Előzmény: [1043] Kata12, 2008-05-20 16:00:04
[1043] Kata122008-05-20 16:00:04

Sziasztok!

Két szerkesztési feladat megoldásában kérném a segítségeteket, amelyet az euklides szerkesztő programban kellene megszerkesztenem

1, Szerkesszünk egyenlő szárú háromszöget, ha adott az alap és a szár különbsége, továbbá a szárak közti szög.

2, Szerkesszünk négyszöget, ha ismert két átlója, az átlók szöge és a két szomszéédos szöge.

Előre is köszönöm a segítséget: Kata

[1042] BohnerGéza2008-05-18 23:09:18

134. egy nehéz feladat:

Adott 3 kör, melyek középpontjai nincsenek egy egyenesen. Adjunk meg olyan inverziót, melynél a körök képe kör marad, de ezek kp-jai már egy egyenesen vannak.

[1041] BohnerGéza2008-05-17 17:27:06
Előzmény: [1034] HoA, 2008-05-05 18:18:51
[1040] szabótimi2008-05-11 16:41:50

Üdvözlet! Ha valakinek van egy kis ideje kérem segítsen hogyan oldható meg ez a számomra bonyolult feladat! Nagyon fontos lenne!Kérlek titeket!Előre is köszi! A feladat: Ábrázoljuk a PQR háromszög köré írt kört. Szerkesszük meg a lényeges átmérők mindkét képét a végpontokhoz tartozó érintőkkel együtt, és állítsuk elő az érintőket a háromszög csúcsaiban is. Rajzoljuk meg a vetületi görbéket. P(45, 140, 265); Q(45, 40, 205); R(145, 140, 205).

[1039] HoA2008-05-08 22:34:33

A piros tetraédet - pontosabban annak B1-ben összefutó három lapját - a kék tetraéder A1BC1 lapja egy szabályos háromszögben metszi. A háromszög oldalai az ABB1A1 , BCC1B1 és A1B1C1D1 lapok középpontjait egymással összekötő három szakasz. Az alakzat szimmetriája miatt ez mind a nyolc csúcsról megállapítható. A közös részt tehát 8 szabályos háromszög határolja. A neve oktaéder. Lásd pl. http://mbuttons.bolyai.hu/upload/VRML/palyazat/kapcs.htm : Egy kocka megfelelő lapátlói szabályos tetraédert alkotnak. A két így kapott tetraéder közös része egy szabályos oktaéder.

Előzmény: [1038] Mate~, 2008-05-08 18:47:50
[1038] Mate~2008-05-08 18:47:50

zsolla feladatának megoldása engem is érdekelne, van rá egy gondolatom, csak 2 pontom helyzete bizonytalan. Tudtok segíteni? Itt egy gyorsan összedobott ábra:

[1037] Mate~2008-05-08 17:47:13

133: Én most hirtelen csak a P-t látom: ABG háromszög egybevágó BCD háromszöggel, hiszen BG=BC, AB=BD és ABGszög= CBDszög (derékszög). Mivel AB merőleges DB-re és BG merőleges BC-re, AG-nek is merőlegesnek kel lennie CD-re. Tehát az APCszög derékszög, így az APC háromszög derékszögű. Thalesz miatt pedig a P pontnak rajta kell lennie a k3 körön. A többin még agyalok. :)

Előzmény: [1034] HoA, 2008-05-05 18:18:51
[1036] zsolla2008-05-08 13:55:43

Keresem a következő feladat megoldását: Az ABCD A1B1C1D1 kocka A C B1 D1 csúcsai és az A1 C1 B D csúcsai is egy-egy tetraédert határoznak meg. Milyen test lesz a két tetraéder közös része?

[1035] BohnerGéza2008-05-06 22:01:00

Érdekesség a 133. feladathoz: Az A-t és C-t rögzítsük! B-t mozgatva AB-n a kapott DR egyenesek átmennek egy közös ponton.

Fogalmazzuk meg a fentinek megfelelő másik állítást is!

Igazoljuk az állítást!

Előzmény: [1034] HoA, 2008-05-05 18:18:51
[1034] HoA2008-05-05 18:18:51

133.feladat-nak javaslom az alábbit: Egy e egyenes 3 pontja ebben a sorrendben A, B és C. Vegyük fel az egyenes egyik oldalán a T középpontú ABDE és az U középpontú BCFG négyzeteket, valamint az AB átmérőjű k1 , BC átmérőjű k2 és AC átmérőjű k3 félköröket.

a) Igazoljuk, hogy az AG és CD egyenesek P metszéspontja k3-ra, az AU és CE egyenesek R metszéspontja k2-re, az AF és CT egyenesek Q metszéspontja k1-re esik.

b) Mi jellemzi a P, Q, R ponthármast?

[1033] BohnerGéza2008-04-29 19:18:38

Az itteni 132. feladat egy - inverzióval történő - megoldása látszik a "Valaki mondja meg!" témában lévő [479]. hozzászólásából. Néhány "elemibb" már itt, az előző hozzászólásokban megszületett. Köszönöm!

Előzmény: [1032] HoA, 2008-04-29 13:28:03
[1032] HoA2008-04-29 13:28:03

Ha az elemibb megoldást a 3 érintkező kört érintő kör szerkeztésére gondolod, egyelőre nem látom az utat. A Geometriai feladatok gyűjteménye az általános Apolloniusz feladatot oldja meg középiskolában tanult módszerekkel:

-az adott és a szerkesztendő körök növelésével-zsugorításával visszavezeti a feladatot két kört (k1,k2) érintő, adott P ponton áthaladó ke kör szerkesztésére - F2 ( elfajuló esetekben egy kör és két pont, illetve 3 pont)

-F2 megoldása során megszerkeszti a körök egyik hasonlósági pontját P-vel összekötő egyenesnek másik, ke-re illeszkedő pontját (Q)

-Végül megszerkeszti a két adott ponton (P, Q) áthaladó, pl. k1-et érintő kört. Ez a megoldás elemibb abban az értelemben, hogy nem használ inverziót, viszont sokkal hosszadalmasabb. És ami a fő baj, nem használja ki azt a speciális helyzetet, hogy a 3 adott kör érinti egymást. F2 megoldása során előjönnek az érintési pontok és a hasonlósági pont közötti kapcsolatok, de nem látom, esetünkben hogyan lehetne alkalmazni.

Előzmény: [1022] BohnerGéza, 2008-04-27 15:01:30
[1030] Gyarmati Péter2008-04-28 21:06:52

Hát igen, a hasonlósági pont :-( Ezt illett volna tudnom. A körrel kapcsolatban másnak is utánanéztem itt.

Előzmény: [1029] HoA, 2008-04-28 17:46:08
[1031] Káli gúla2008-04-28 19:34:24

Jelölje az átmérőgyanús pontokat P és Q. A PQ ívet a két érintési pont, A, B három körívre bontja, az ezekhez tartozó kerületi szögek legyenek \alpha, \gamma és \beta. Ekkor a CPQ háromszög szögösszege 2\alpha+2\beta+2\gamma, mert az e közös érintő a C-nél lévő szöget a bal oldali ábrán (\alpha+\gamma) és (\beta+\gamma) részre, a jobb oldali ábrán \alpha és \beta részre osztja (a sima és az elfajult kerületi szögek egyenlőségét kell a közös érintőknél figyelembe venni). Tehát 2\alpha+2\beta+2\gamma=180o, azaz a három körív együtt egy félkört alkot.

Elnézést a többszörös beírásért, az előző három, képtelen hozzászólásomat ki is lehetne törölni. Ha egy moderátor olvassa, köszönöm előre is. (törölve - Sirpi)

Előzmény: [1020] BohnerGéza, 2008-04-27 03:22:52
[1029] HoA2008-04-28 17:46:08

Az állítás abból adódik, hogy két érintkező kör érintési pontja egyben a két kör kölső vagy belső hasonlósági pontja is. Talán jobban áttekinthető kívülről érintkező körökre, ekkor az érintési pont belső hasonlósági pont. Az ábrán a hasonlóság miatt PO3||T12O1ésQO3||T12O2. Mivel O1,O2ésT12 egy egyenesen vannak, ezért P,QésO3 is, vagyis PQ a kör átmérője.

Előzmény: [1019] BohnerGéza, 2008-04-27 02:52:47
[1025] Gyarmati Péter2008-04-28 17:01:56

Igen, látom. Például a \varphi3 esetében: ha a kerületi és középponti szögek összefüggését vizsgálom, akkor AB-ról még nem feltételeztem, hogy átmegy O-n. Viszont amikor az E1BO háromszögben az alapon fekvő szögekről állítom, hogy \varphi3, akkor már igen. Ellenben úgy tűnik, O1O2 párhuzamos az OB-vel.

[1024] Gyarmati Péter2008-04-28 16:36:16

Nem használtam fel azt, hogy az E érintési pont. Lehet, hogy felhasználtam a bizonyításban a bizonyítandó állítást?!

Így túl elemi volna...

[1023] Gyarmati Péter2008-04-28 16:16:48

Azt kell igazolni, hogy az A, O és B pontok egy egyenesen vannak. Elég belátni, hogy AE1B =90 (E1 az AB Thalesz körén van, tehát AB egy átmérő).

Először a kerületi és középponti szögek összefüggéseit, valamint az egyenlő szárú háromszögek alapon fekvő szögeinek egyenlőségét használjuk ki. Ezzel adódnak az ábrán látható szögek.

Az AOE2 háromszögben: 2\varphi1+2\varphi2+2\varphi3=180, tehát \varphi1+\varphi2+\varphi3 = 90.

AE1B = (\varphi1+\varphi2)+\varphi3= 90, így az állítást igazoltuk.

Előzmény: [1020] BohnerGéza, 2008-04-27 03:22:52
[1022] BohnerGéza2008-04-27 15:01:30

A feladat Hoa a "Valaki mondja meg!" témában lévő [479]. hozzászólásából adódott. Remélem "elemibb" megoldás is születik rá!

Előzmény: [1021] jonas, 2008-04-27 12:31:20
[1021] jonas2008-04-27 12:31:20

Ez meglepő.

Előzmény: [1020] BohnerGéza, 2008-04-27 03:22:52
[1020] BohnerGéza2008-04-27 03:22:52

Ábra a 132. feladathoz:

Előzmény: [1019] BohnerGéza, 2008-04-27 02:52:47
[1019] BohnerGéza2008-04-27 02:52:47

132. feladat: Három kör páronként érinti egymást három, nem egy egyenesen lévő pontban. Két kör érintési pontján át húzzunk egy-egy egyenest a másik két ponton át! Bizonyítsuk be, hogy ezeknek a harmadik körrel való másik metszéspontja a kör két átellenes pontja!

[1018] HoA2008-04-21 09:25:28

Legyen A'B'C' az ABC háromszög képe az affinitásnál. Ekkor A'B'C' oldalegyenesei is a P, Q, R pontokban metszik a t tengelyt. Az affinitás párhuzamosság-tartró tulajdonsága miatt C'S és A'R párhuzamosak. Ezért A'B'C' szög és B'C'S szög váltószögek, egyenlőek. Így C' pontból a QS szakasz az egyik adott (béta), PQ pedig a másik adott - én C'-nél lévő szög miatt gammának nevezném - szög alatt látszik. Ebből a leírt szerkesztés már adódik.

Előzmény: [1014] HoA, 2008-04-20 08:50:18
[1017] t.balint89112008-04-20 15:09:26

Köszönöm!

[1016] cocka2008-04-20 11:36:02

Köszönöm szépen.

Kijött. :)

Előzmény: [1014] HoA, 2008-04-20 08:50:18
[1015] HoA2008-04-20 08:58:50

Ha geometriai feladatról van szó, vegyük fel a Q pontot tartalmazó, PQ szakaszra merőleges S síkot. Ebben a síkban rajzoljunk egy Q középpontú, 35 mm oldalú négyzetet. A négyzet csúcsai és P alkotják a keresett gúlát.

Ha ábrázoló geometria, akkor javaslom, tedd fel abba a fórumba.

Előzmény: [1012] t.balint8911, 2008-04-19 13:25:19
[1014] HoA2008-04-20 08:50:18

Kedves Cocka!

Egyelőre, hogy ne bosszankodj tovább, a válasz kérdésedre: az adott látószögű köríveket PQ és QS fölé kell rajzolni. Itt most nincs rajzoló programom, úgyhogy ábra és indiklás holnap.

Előzmény: [1013] cocka, 2008-04-20 00:42:09
[1013] cocka2008-04-20 00:42:09

Nekem is lenne egy érdekes kérdésem a kedves matematikát szerető kollégákhoz.

Konkrétan egy geometriai feladatról van szó, azonbelül is egy tengelyes affinitásról.

A szövege a következő:

Adott a tengelyes affinitás tengelye és egy ABC háromszög. Határozzuk meg az affinitást úgy, hogy az A'B'C' hasonló legyen egy előre megadott DEF háromszöghöz.

Hát ugye az ember próbálkozik, de sajnos nekem nem jött össze. A megoldás szerint a szerkesztés lépései a következők:

Megrajzolom a t tengelyt. Megrajzolom az ABC háromszöget úgy, hogy az oldalait meghosszabbítom. CA P-ben, CB Q-ban, AB R-ben, az AB-vel a C-n keresztül húzott párhuzamos egyenes pedig S-ben metszi a tengelyt.

Az ABC háromszög mellé valahova megrajzolom a DEF háromszöget. Az F szögét elnevezem alfának, az E szögét meg bétának. A megoldás azt javasolja, hogy a PQ szakasz fölé (alá) szerkesszünk az alfa szöggel látószög körívet, illetve az RS fölé (alá) is szerkesszünk egy látószög körívet a béta szöggel. Na most elvileg a látószög körívek metszéspontja fogja kiadni a C' helyét. Ekkor ugye a feladat tovább úgy folytatódik, hogy a C'-t összekötöm a C-vel és ez lesz az affinitás iránya.

Igen ám, na de akármilyen helyzetű az ABC háromszög ez a két kör így a büdös életben nem fogja metszeni egymást. (az sem mellékes egyébként, hogy a megoldásban ez az egész feladat 3 sorban el van intézve a szerkesztési lépéseket erősen mellőzve, sőt ábra sincs, nehogy már megértsd :( )

Szóval a kérdésem végül is az lenne, hogy mely szakaszok fölé is kéne a látószög köríveket szerkeszteni, mert a PQ és RS páros biztosan nem nyerő. Próbáltam úgy is, hogy PR és QS, így metszi a két körív egymást, de a kapott háromszög rohadtul nem hasonlít a DEF háromszögre, mert pl. a szögeik kapásból nem egyeznek meg. Én meg valami olyasmit hallottam a hasonló háromszögekről, hogy szögei páronként megegyeznek.

A feladattal bohóckodtam egy sort Geogebrában, aztán hagytam az egészet a francba. Akinek van valami ötlete ne kíméljen. Akár privátban is. Thanks.

[1012] t.balint89112008-04-19 13:25:19

Tisztelt forumozók !

Nem tudom hogyan áljak neki ennek a feledatnak:

Adott a PQ szakasz . Szerkeszen egy olyan szabályos négyoldalú gúlát , amelynek P a csucspontja, Q pedig a 35 mm élű alapnégyzetnek a középpontja !

[1011] akinom912008-04-15 12:39:54

Azt hiszem ez volt az, koszonom szepen :)

Előzmény: [1010] sakkmath, 2008-04-15 11:47:19
[1010] sakkmath2008-04-15 11:47:19

A Ptolemaiosz-tétel általánosabb változata:

Ha egy négyszög oldalai a, b, c és d, átlói e és f, akkor ac+bd \ge ef, és egyenlőség akkor áll, ha a négyszög húrnégyszög. (Az egyenlőtlenség bal oldalán a szemközti oldalak szorzatának összege áll.)

Előzmény: [1009] akinom91, 2008-04-15 11:22:48
[1009] akinom912008-04-15 11:22:48

Volt egy tetel, mar nem is tudom pontosan kirol neveztek el (talan Descartes, de nem vagyok biztos benne), amely szerint egy negyszog atloinak paronkenti szorzata egyenlo, vagy legalabbis valami ilyesmi. Tudna valaki segiteni a pontos megfogalmazasaban, alkalmazasaban?

[1008] Káli gúla2008-04-15 00:15:26

Legyen az ABCD tetraéder AB élén atmenő szögfelező síkról szó., messe ez a CD-t E-ben. Húzzunk C-n és D-n át (c és d) párhuzamos egyeneseket az AB szakasszal. A c és d által meghatározott sík messe a szögfelező síkot e-ben. Legyen még az AB egyenes neve l. Ezzel a négy párhuzamos egyenessel (l, c, d, e) tulajdonképpen visszavezettük a két dimenziós kérdésre a feladatot:

\frac{d(C,E)}{d(D,E)}=\frac{ d(c,e)}{d(d,e)} =
\frac{ d(c,l)}{d(d,l)}= \frac{ m_C}{m_D} = \frac{ m_C \cdot AB}{m_D\cdot AB} = \frac{ T(ABC)}{T(ABD)}

(A második egyenlőségnél használtuk a kétdimenziós tételt, abban a síkban, amelyik a négy párhuzamosra merőleges.)

Előzmény: [1006] Gyöngyő, 2008-04-14 22:45:39
[1007] Róbert Gida2008-04-14 23:27:25

Ez akkor annak a tételnek az általánosítása, hogy egy háromszögben a szögfelező a szemköztes oldalt a szomszédos oldalak arányában osztja.

Előzmény: [1006] Gyöngyő, 2008-04-14 22:45:39
[1006] Gyöngyő2008-04-14 22:45:39

Sziasztok!

Tudnátok segíteni a következő feladatnál:

Bizonyítsa be,hogy a tetraéder lapszögfelező síkja olyan arányban osztja az átellenes élet,amely egyenlő annak a két lapnak a területarányával,amelyek szögét felezi!

Köszike:

Zsolt

[1005] BohnerGéza2008-04-13 20:22:04

Legyen a 131. feladatban H ill. J az A'B'-nek B'-höz ill. C'D'-nek D'-höz közelebbi harmadolópontja.

Igazoljuk, hogy H, B, D és J egy egyenesen vannak!

Adjuk meg a HB, a BD és a DJ szakaszok hosszának arányát!

Előzmény: [1004] HoA, 2008-04-03 11:20:44
[1004] HoA2008-04-03 11:20:44

A 131. feladat megoldása:

a ) I.) A csúcspontok helyvektoraira - AB' felezőpontja B - felírható: B = \frac{A + B'}2 B'=2.B-A , hasonlóan C'=2.C-B , D'=2.D-C és A'=2.A-D . Az egyenletrendszert pl. D-re megoldva kapjuk:

D = \frac1{15} \cdot (8D' + 4C' + 2B' + A') Ha koordinátarendszerünk kezdőpontjának D'-t választjuk, akkor \vec{D'} = \vec0 , A' = \vec{D'A'}, B' = \vec{D'B'} , C' = \vec{D'C'} ,

\vec{D'D} = \frac1{15} \cdot ( 4 \vec{D'C'} + 2\vec{D'B'} + \vec{D'A'}) , ami A'B'C'D' négyszöget ismerve könnyen szerkeszthető.

II.) Szemléletesebb megoldást kapunk, ha - mint BohnerGéza is a kiegészítésben utalt rá - a feladatban kínálkozó középpontos hasonlóságokat nézzük. Legyen H1 a \lambda1=1/2 arányú, A' középpontú hasonlóság. Ez D-t A-ba képezi le. A H2,H3ill.H4 , mind \lambda2=\lambda3=\lambda4=1/2 arányú, B' , C' ill. D' középpontú hasonlóságok A-t B-be, B-t C-be, ill. C-t D-be képezik le. A négy hasonlóság eredője (szorzata) H5=H1.H2.H3.H4 tehát D-t fixen hagyja. Mivel \lambda1.\lambda2.\lambda3.\lambda4=1/16\ne1,H5 is egy középpontos hasonlóság, amelynek csak a középpontja fixpont, tehát D H5 középpontja. Felhasználva, hogy adataink alapján H1,H2,H3ésH4 a sík tetszőleges pontjára végrehajtható, valamint hogy középpontos hasonlóságnál egy pont, a képe és a hasonlóság középpontja egy egyenesen van, az alábbi szerkesztés adódik:

Vegyünk fel egy P0 és egy Q0 pontot. P0 képe H1-nél P1 , P1H2-nél P2, P2H3-nál P3, végül P3H4-nél P3. Hasonlóan kapjuk a Q1,Q2,Q3,Q4 pontokat. P0P4 és Q0Q4 egyenesek metszéspontja D.

P0 és Q0 ügyes megválasztásával a lépések száma csökkenthető. Például legyen P0=A'ésQ0=B' . Ekkor P1 is A', P2 A'B' felezőpontja, P3P2 és C' felezőpontja, P4 pedig P3 és D' felezőpontja. Q1 A'B' felezőpontja ( =P2) , stb . ld az ábrán.

Tovább egyszerűsíthető a szerkesztés, ha figyelembe vesszük, hogy H5 aránya 1/16, így D-t csak P0 és képei alapján is megkaphatjuk: a P0P4 szakaszt hosszabítsuk meg P4-n túl az 1/15-ével.

Szerkesztésünk helyességét igazolja ha a lépéseket számítással követjük. Tetszőleges P0-ból indulva P_1 = (P_0+A')/2 ; P_2 = (P_1 + B')/2 ... P_4 = 1/16 ( P_0 + A' + 2B' + 4 C' + 8D') ; D = P_0 + 16/15 ( P_4 - P_0 ) = \frac1{15} \cdot (8D' + 4C' + 2B' + A') , mint azt az egyenletrendszerből kaptuk.

b) Az AC átlóval kettévágott négyszög egyik része , az ABC \Delta területe fele a BB'C' \Delta -nek, mivel AB = BB' és C'B = 2 CB. Hasonlóan a négyszög másik része, ACD \Delta területe fele az A'DD' \Delta -nek. E két \Delta területe tehát együtt a négyszög területének duplája. Ugyanez igaz a BD átlóval kettévágott négyszögre és az A'B'A ill. C'D'C \Delta -ekre. Így A'B'C'D' területe 5-szöröse az ABCD területének.

Előzmény: [1002] BohnerGéza, 2008-03-27 22:32:12
[1003] BohnerGéza2008-04-02 19:41:53

Az a.) rész mo-hoz leképezések szorzatát ajánlom!

Előzmény: [1002] BohnerGéza, 2008-03-27 22:32:12
[1002] BohnerGéza2008-03-27 22:32:12

131. feladat: Az ABCD pozitív körüljárású négyszög minden oldalát, pozitív körüljárást tartva, meghosszabbíttottuk az oldal hosszával, kaptuk az A'B'C'D' négyszöget. ( pl. az AB' felezőpontja B )

a.) Ismerve A'B'C'D' négyszöget, szerkesztendő az ABCD!

b.) Hányszorosa az A'B'C'D' területe az ABCD területének?

[1001] HoA2008-02-18 15:56:38

A 130.feladat megoldása: Legyen az x, y, z szakaszok talppontja az a, b, c oldalak egyenesén rendre Ta,Tb,Tc. Merőleges szárú szögekről lévén szó, x és y bezárt szöge \gammavagy\pi-\gamma, mindenesetre a PTaTb \Delta területe \frac12 \cdot x \cdot y \cdot sin \gamma . Hasonló igaz PTbTc és PTcTa \Delta -ekre. Kifejezésünkben, melyet jelöljünk F-fel, helyettesítsünk a=2Rsin\alpha, b=2Rsin\beta, c=2Rsin\gamma szerint, ekkor F=2R(y.z.sin\alpha+z.x.sin\beta+x.y.sin\gamma). A zárójelben a fenti \Delta-ek előjeles területösszegének kétszerese áll. A továbbiakban azt vizsgáljuk, ez P milyen helyzetére lesz 0.

A \Delta belsejében biztosan nem, hiszen ott mindhárom tag pozitív. A \Delta 3 csúcsa viszont megfelel, hiszen ott x,y,z közül kettő 0, így az összeg mindhárom tagja 0. A 3 egyenes által 7 részre vágott síknak abban a 3 részében, melyek határán csak egy hsz csúcs van, x,y,z közül 2 negatív és 1 pozitív (P1 pont) , az előjeles \Delta területekből is így 2 negatív és 1 pozitív, de a két negatív területű \Delta egyesítése magában foglalja a pozitív területűt, így az összeg nem lehet 0. ( Ezt persze bizonyítani kell ) . A maradék 3 síktartományban x,y,z közül 2 pozitív és 1 negatív (P2 pont), az előjeles \Delta területekből így 2 negatív és 1 pozitív, az előjeles összeg csakkor lesz 0, ha a \Delta területek abszolút értékei közül kettőnek az összege megegyezik a harmadikkal. Ez pedig csakkor teljesül, ha a TaTbTc \Delta területe 0, vagyis ha Ta,Tb,Tc egy egyenesbe esik. Mivel ezek éppen a P-ből az oldalegyenesekre bocsátott merőlegesek talppontjai, a keresett mértani hely a \Delta körülírt köre.

UI: Itt is megkérdezem, tudja-e valaki, mi lett a sulinet.hu KöMaL archívumával?

Előzmény: [999] BohnerGéza, 2008-02-10 11:50:42
[1000] BohnerGéza2008-02-10 12:05:11

Ábra a 130. feladathoz:

Előzmény: [999] BohnerGéza, 2008-02-10 11:50:42
[999] BohnerGéza2008-02-10 11:50:42

130. feladat: Legyen a P pont előjeles távolsága az ABC háromszög oldalaegyeneseitől rendre x, y ill. z.

( Pl. x pozitív, ha BC-nek az A felöli oldalán van. )

Adjuk meg azon P-k mértani helyét, melyekre ayz + bzx + cxy = 0!

[998] HoA2008-01-31 13:04:36

Igen, szerintem nagyon szép "szerkesztéses" megoldás. Ha már itt tartunk, nem tudom volt-e már a fórumom - vagy máshol - a "szerkesszünk háromszöget ha adottak a magasságvonalai" feladat. És persze nem a reciprok szakaszok szerkesztésére, vagy - ha létezik - a magasságvonalakból szerkesztett háromszög magasságvonalaira, mint a szerkesztendővel hasonló háromszögre gondolok.

Előzmény: [997] BohnerGéza, 2008-01-30 15:29:05
[997] BohnerGéza2008-01-30 15:29:05
Előzmény: [996] BohnerGéza, 2008-01-30 14:29:12
[996] BohnerGéza2008-01-30 14:29:12

Nem tudom "szerkesztésesebb" megoldás-e, mint HoA-é a [993]-ban.

( Az Oa-t megkapjuk,ha O-t L-re tükrözzük. )

Előzmény: [992] komalboy, 2008-01-26 17:23:57
[995] HoA2008-01-29 17:59:48

Talán egy egylépéses meggondolás segít. Húzzuk meg a körnek a háromszög oldalaival párhuzamos érintőit. Mivel a háromszöglemez tartalmazza a kört, ezek az érintők nem alkothatnak az eredetinél nagyobb háromszöget, legfeljebb kisebbet. Ezt a második háromszöget a körrel együtt nagyítva az eredeti háromszöget és annak beírt körét kapjuk. A beírt kör tehát nem lehet kisebb eredeti körünknél.

Előzmény: [994] tolgyesik, 2008-01-29 17:20:34
[994] tolgyesik2008-01-29 17:20:34

A körül írt kör esetén én is megtaláltam az ellenpéldát, de a beírt kör esetén még mindig bizonytalan vagyok.

[993] HoA2008-01-29 10:48:47

Gondolatébresztőnek, amíg nem születik egy igazi "szerkesztéses" megoldás. Legyen adott az O középpontú, R sugarú k körvonalon az A pont, a kör belsejében a Q pont. Az Euler tétel szerint d2=R(R-2r) . Esetünkben d = OQ . Legyen az OQ-t Q-ban érintő, k-t metsző k' kör és k egyik metszéspontja S, OS és k' másik metszéspontja T. Ekkor a szelőtétel értelmében d2=OQ2=OS.OT=OS(OS-ST)=R(R-ST) . Tehát ST = 2r. A Q körüli, ST átmérőjű kör a háromszög beírt köre, a B és C csúcsokat e kör A-ból húzott érintői metszik ki k-ból.

Előzmény: [992] komalboy, 2008-01-26 17:23:57
[992] komalboy2008-01-26 17:23:57

Egy (remélem) könnyed feladat a geometriakedvelőknek: Adott egy körvonalon egy pont, és egy pont a körvonalon belül. Határozzunk meg a körvonalon másik két pontot úgy, hogy a körvonalra illeszkedő, ezen három pont alkotta háromszög beírható körének középpontja legyen az adott belső pont.

[991] rizsesz2008-01-25 13:27:31

Amúgy amit 7-re írtam, az igaz :)?

Előzmény: [989] rizsesz, 2008-01-25 13:11:46
[990] jonas2008-01-25 13:24:05

A 8. viszont nem igaz általában. Hegyesszögű háromszögre a körülírt kör a legkisebb sugarú ilyen kör, tompaszögű háromszögre viszont a hosszú oldal Thales-köre az -- derékszögűre a kettő megegyezik.

Előzmény: [988] tolgyesik, 2008-01-25 11:52:31
[989] rizsesz2008-01-25 13:11:46

7. egy olyan kör van, ami egy háromszög minden oldalát belülről érinti, ez a beírt kör. Tegyük fel, hogy találtunk egy maximális sugarú kört, ami nem érinti mind3 oldalt. Ekkor ha 2 érintési pont van, akkor azokat rögzítve a kört nagyíthatjuk, méghozzá úgy, hogy a középpontját a 2 két érintési pont szakaszfeleő merőlegesén eltoljuk. Így előbb-utóbb mindenképpen eljutunk a beírt körhöz.

Előzmény: [988] tolgyesik, 2008-01-25 11:52:31
[988] tolgyesik2008-01-25 11:52:31

Kedves Fórumozók!

Két régi feladatról olvastam a fórumon, és felmerült bennem a kérdés, hogy ez a két állítás mégsem triviális?

7. feladat: Igaz-e, hogy a (nem elfajuló) háromszög beírt köre a legnagyobb sugarú kör, melyet a háromszöglemez tartalmaz?

8. feladat: Igaz-e, hogy a (nem elfajuló) háromszög köré írt kör a legkisebb sugarú kör, mely tartalmazza a háromszöglemezt?

[987] Süni1312008-01-23 20:48:01

Kedves Fórumozók!

Egyező átmérőjű körök középpontjai egy egyenesen, egymástól egyenlő távolságra helyezkednek el. Hogyan tudnám meghatározni egy másik egyenesnek az előzővel bezárt minimális szögeit, ha a feltétel a következő:

az egyenes által kimetszett húrhosszúságok összege nem haladhatja meg az 1, 2,...,n körátmérőt, az egyenesnek az adott szögek alatti bárhová történő eltolása mellett.

A segítséget előre is köszönöm,

Üdv: Süni131

[986] HoA2008-01-21 14:03:33

Konkrét irodalmat nem ismerek, de mivel eddig senki sem reagált, leírom az ötleteimet. Szerintem a feladatot a regressziós egyenes mintájára lehet kezelni. Ott adva van n db pont - (xi,yi) koordinátapár - melyek nem pontosan egy egyenesre illeszkednek és a feladat a pontokra legjobban illeszkedő y = ax + b egyenes megadása. Mint tudjuk, ha az eltérés mértékének a \sum {\Delta y^2} -t vesszük, ahol \Deltayaz egyenes xi -beli y koordinátájának és yi -nek a különbsége, akkor az eltérés a ill b szerinti deriváltját 0-nak véve az egyenes a ás b paramétereire elsőfokú egyenletrendszert kapunk.

Egyszerűség kedvéért vegyünk először egy síkbeli példát 3 ponttal. Legyenek adva az A,B,C pontok, valamint az A*, B*, C* pontok koordinátái. Feltesszük, hogy az ABC és A*B*C* háromszögek nagyjából hasonlóak és keressük azt a transzformációt, mely ABC-t az A*B*C*-t jól közelítő A'B'C'-be viszi át. Ha AB és A'B' különböző hosszúak és nem párhuzamosak, ez egy nyújtva forgatás. Ha alakzatainkat a komplex számsíkon ábrázoljuk, a nyújtva forgatásnak egy z'=z0+wz transzformáció felel meg, - ld. Pl. Reimann István:Geometria és határterületei - és itt is az a feladat, hogy határozzuk meg a z0 és w számokat úgy, hogy az A,B,C pontok képei, A'B'C' a "legközelebb" legyenek az A* B* C* pontokhoz. A hiba mértékének talán itt is tekinthetjük az A'A* ... távolságok négyzetösszegét. Azt persze nem mondom, hogy z0-ra és w-re itt is elsőfokú egyenletrendszert kapunk, de ha máshogy nem, közelítő módszerekkel a feladat megoldható.

Térbeli transzformációnál még bonyolultabb a helyzet, de ott is megtalálható az a transzformáció, mely két hasonló alakzat egyikét a másikba átviszi (tenzor?) és ott is felírható a megadott A*, B* , ... és a transzformáció által létrehozott A', B', ... pontok távolságának négyzetösszege, illetve ennek az összegnek a transzformációs objektum paramétereitől való függése. Utána már csak néhány deriválás és egyenletrendszer megoldás van hátra..

Előzmény: [983] farkasb, 2008-01-08 19:34:30
[983] farkasb2008-01-08 19:34:30

Tisztelt Fórumozók!

Nem egy konkrét feladatot, kérdezek, hanem csak azt, hogy ismer-e valaki olyan segédletet, anyagot, módszert, ami alapján el tudnék készíteni egy térbeli transzformációt, hogy: - adott egy térbeli alakzat elméleti alakjának koordinátái egy globális rendszerben - adott ugyanennek az alakzatnak a koordinátái egy lokális koordinátarendszerben, és a pontjai kis mértékben eltérnek az elméleti alakzattól - és ezt a lokális rednszert szeretném a globálisba transzformálni közös pontok felhasználásával úgy, hogy közben lehessen látni az ellentmondásokat, eltéréseket, hibákat, és ki lehessen venni a transzformációs paraméterek számításából azokat a pontokat, amik nagy mértékben eltérnek. Előre is köszönnettel: farkasb

[982] BohnerGéza2008-01-06 13:56:35

Kettővel ezelőtti hozzászólásomban E-t és F-et végig felcseréltem. Elnézést!

Előzmény: [984] BohnerGéza, 2008-01-06 02:15:02
[981] BohnerGéza2008-01-06 02:29:39

Érdemes tudni, hogy egy AB szakasz látókörének két íve nem teljesen egyenértékű. Ha az egyikről AB fí szögben látszik, akkor a másikról -fí-ben.

Tekintsük HoA [978] ábráját! Ott lényeges a feladat szempontjából, hogy F-ből DA és CB ugyanolyan irányítású egyforma szögben látszik, ez a DEA(=CEB) szöggel egyenlő.

Előzmény: [979] HoA, 2008-01-04 15:42:37
[984] BohnerGéza2008-01-06 02:15:02

Köszönöm HoA!

A 129. feladat kitűzésekor valóban arra gondoltam, ha minden olyan ABCD négyszögre igaz, melyben AB nem egyenlő CD-vel, azaz valódi hasonlósággal (körüljárástartó) kaphatjuk AB-ből CD-t, akkor E a megfelelő forgatva nyújtásnsak a kp-ja, tehát a fixpontja is.

Ehhez minden lehetőségre meg kell mutatni, hogy E mindig a körök AFD ill. BFC ívén van. (Pl. akkor is, ha F az AB szakaszon van. )

Még azzal is, kell kezdeni valamit, ha a két kör érinti egymást - F-ben.

Sőt Az AB párhuzamos CD-t is vizsgálni kell. Mindezt a fórumon pontosan leírni nem érdemes - a következő hozzászólásban egy dologról még írok-, de:

Ha mindent megmutattunk, akkor bebizonyítottuk, hogy minden valódi hasonlóságnak van fixpontja. A feladat - a speciális eseteket kivéve - lehetőséget mutat a fixpont szerkesztésére.

Előzmény: [977] HoA, 2008-01-04 10:43:57
[980] Bubóka2008-01-04 18:36:09

Igen, ezt így tudtam én is, de nem ez volt a feladat, amit nem tudtam. De azért köszi. Hidd el a másik is lehetséges, hisz az egyetemen kérik. De ha megtudom, közre adom!!

Előzmény: [979] HoA, 2008-01-04 15:42:37
[979] HoA2008-01-04 15:42:37

Na látod, ez az, amiről eddig szó sem volt: adott a szakasz felezőponja is. Egy megoldás: P az AS egyenes szabadon választott pontja, PN és SB metszéspontja Q, AQ és PB metszéspontja R, SR a keresett egyenes.

Hogy "adott pontból adott pontra állítson merőlegest" azt meghagyom neked.

Előzmény: [978] Bubóka, 2008-01-04 13:46:22
[978] Bubóka2008-01-04 13:46:22

Tisztelt Fórumozók! Látom galibát okoztam, bár nem állt szándékomban. A feladatom amit nem tudok megoldani, az az volt és nem több, hogy adott pontból adott pontra állítson merőlegest. sajnos nincs itt semmiféle körkp és egyebek. Eljutottunk a párhuzamos szerkeszthetősége vonalzóval vitájához, amit konkrétan nem fejtettem ki, mert nem ez volt a feladatom, pusztán reagáltam Jónásnak arra, hogy lehetséges. De most megteszem. A feladat: adott egy AB szakasz, annak N felező pontja és S pont mely nem illeszkedik a szakaszra. Húzzon S-en át párhuzamost az AB szakasszal. Mivel a (csak)vonalzós szerkesztésekhez 4 adott pontra van szükség így egy 4. pontot tetszőlegesen veszünk föl (az A és S pontot összekötő egyenesen). HA több időm lesz megpróbálom itt megszerkeszteni de legalábbis leírni a menetét, de épp geo. vizsgára készülök.

[977] HoA2008-01-04 10:43:57

Az ábra szerinti elrendezésben az ABF és CDF háromszögek B-nél ill. C-nél lévő szögei (piros) a k2 körben, A-nál ill. D-nél lévő külső szögei (zőld) a k1 körben az EF húrhoz tartozó kerületi szögek. Úgy vélem, BohnerGéza nem a nehézsége miatt adta fel ezt a feladatot, hanem valamilyen érdekes észrevétele van - talán a forgatva nyújtással vagy a négy háromszög tételével kapcsolatban?

Előzmény: [966] BohnerGéza, 2008-01-02 22:51:56
[976] HoA2008-01-04 10:28:35

Én meg úgy gondolom, hogy egy geometria fórumon a szerkesztési feladatokat szabatosan illik megfogalmazni. Mi adott és mit kell szerkeszteni? Ha például kiderül, hogy az egyenesen és ponton kívül még egy kör meg a középpontja is adott, akár merőlegest is szerkeszthetsz az adott ponból az adott egyenesre csak vonalzóval.

Előzmény: [971] Bubóka, 2008-01-04 08:23:54
[975] rizsesz2008-01-04 10:27:45

Elnézést, legközelebb nem csak az egy hozzászólást nézem meg. :) Ennek így valóban előfeltétele egy és más. Bubókának drukkolok a vonalzóval szerkesztéshez!

Előzmény: [974] jonas, 2008-01-04 09:29:08
[974] jonas2008-01-04 09:29:08

Igen, de ott meg van adva egy párhuzamos, és egy másikat kell szerkeszteni. Azt tényleg meg lehet csinálni.

Adott a két párhuzamos fekete egyenes, és a fekete pont. Meghúzod tetszőlegesen a két piros egyenest, az egyiket a fekete ponton keresztül. Utána meghúzod a három narancssárga egyenest a megfelelő metszéspontokat összekötve, utána a zöld egyenest, majd a kéket, végül a rózsaszínt. A rózsaszín párhuzamos lesz a két feketével.

(Nem vagyok benne biztos, hogy ez a legegyszerűbb szerkesztés, lehet, hogy egy egyenest meg lehet spórolni.)

Ha viszont nincs másik párhuzamos adva, csak egy fekete egyenes és egy pont, akkor csak egyenes vonalzóval szerintem nem lehet megszerkeszteni a rózsaszín párhuzamost. A 969. hozzászólást egyszerűen nem gondoltam át.

Előzmény: [972] rizsesz, 2008-01-04 08:57:56
[973] nadorp2008-01-04 09:20:01

Az a feladat is így kezdődik: "Adott egy trapéz,...", azaz nem csak egy egyenes és egy pont adott síkon,hanem egy párhuzamos egyenespár és egy pont. Jonas épp azt kérdezte [969], hogy Bubóka eredeti feladatában nincs-e más is megadva a ponton és az egyenesen kívül.

Előzmény: [972] rizsesz, 2008-01-04 08:57:56
[972] rizsesz2008-01-04 08:57:56

Jaja, bár nincsen ott a megoldásnál, de a 2001/2002 február B. 3527. is ezen alapult.

Előzmény: [971] Bubóka, 2008-01-04 08:23:54
[971] Bubóka2008-01-04 08:23:54

Elnézést lehet kérni, de lehet párhuzamost húzni vonalzóval!!!!!! Adott ponton át adott egyenesre, "trapéz feladat" segítségével! Én úgy gondolom, hogy azért mert valaki valamit nem tud az nem egyenlő azzal, hogy olyan nem létezik.

Nem tévedés volt a merőleges. Biztos hogy lehet, csak én nem jövök rá egyenlőre a menetére.

Előzmény: [970] jonas, 2008-01-03 22:17:18
[970] jonas2008-01-03 22:17:18

Párhuzamost csak vonalzóval szintén nem lehet. Elnézést.

Előzmény: [969] S.Ákos, 2008-01-03 21:21:58
[969] S.Ákos2008-01-03 21:21:58

Így hirtelen nem látom, hogy párhuzamost hogy lehetne csak vonalzóval szerkeszteni. Felvilágosítanál?

Bubóka: Az 5-szög szerkesztése meg abból következik, hogy az egységnyi sugarú körbe írt 10szög oldala \frac{\sqrt5-1}2. Ez pithagorasz-tétel segítségével meg könnyen szerkeszthető.

Előzmény: [968] jonas, 2008-01-03 19:31:17
[968] jonas2008-01-03 19:31:17

"Hogy kell pontból egyenesre merőlegest bocsátani CSAK vonalzóval?"

Nem párhuzamosra gondolsz véletlenül? Ha nem, akkor nincs valami más is adva? Mert így lehetetlen.

Előzmény: [967] Bubóka, 2008-01-03 18:22:34
[967] Bubóka2008-01-03 18:22:34

Üdv Mindenkinek! Segítségeteket szeretném kérni. Hogy kell pontból egyenesre merőlegest bocsátani CSAK vonalzóval? HA a menetét valaki leírná, nagyon megköszönném. Ja! És szabályos ötszöget szerkeszteni az aranymetszéssel...?

[966] BohnerGéza2008-01-02 22:51:56

129. feladat: Az ABCD négyszög nem trapéz. AB és CD az E-ben metszik egymást. Az ADE és BCE körülírt köre E-n kívül F-ben találkoznak. Bizonyítandó, hogy ABF és CDF hasonlóak!

[965] S.Ákos2007-12-31 15:13:16

Akkor az elemi megoldás (mik jutnak az ember eszébe hajnali fél három táján): Legyen BD felezőponja E. ekkor DEC\angle=2\alpha, mivel a E a DCB háromszögben a körülírható kör kp-a. De így CDE\angle=CAD\angle=2\alpha, így DCA egyenlőszárú, így \frac{AC}{DB}=\frac12 Legalábbis sztem ez bizonyos esetekben igaz. Ha DCA háromszögben az említett szögek külső szögek, akkor is igaz, hogy DCA egyenlőszárú.

Előzmény: [964] BohnerGéza, 2007-12-31 14:05:01
[964] BohnerGéza2007-12-31 14:05:01

A megkötés valóban nem kell, a kitűzők talán a 9-eseknek szóló feladatot "nehezítették", lehessen általánosítani. A korosztálytól nem feltétlenül trigonometriát használó megoldást vártak. Az elemi tetszett nekem jobban.

Előzmény: [963] SmallPotato, 2007-12-30 19:30:47
[963] SmallPotato2007-12-30 19:30:47

A megoldás szerintem is helyes. (Pontosabban: nekem is ez jött ki. :-) )

Két hozzáfűznivalóm lenne azért:

A feladat kifejezetten hegyesszögű háromszöget ír; Te nem ilyet rajzoltál, bár elsőre nekem sem tűnik lényeginek a megkötés. (Majd lehet, hogy engem is helyreigazítanak. :-D)

A másik: a jövőre nézve szerencsésebb lenne (mivel megszokott), ha a háromszög csúcsait az óramutató járásával ellentétes sorrendben betűznéd, és a szögeket ugyanezen sorrendben osztanád ki (az A csúcsban \alpha, a B csúcsban \beta stb.)

Azért merem ezt kérni, mert emlékszem első táblai geometria-szereplésemre a gimiből: nem szokványosan betűztem a háromszöget, és a padsorokból tömény húúúúúú jött ... :-)))

Előzmény: [962] S.Ákos, 2007-12-30 12:06:40
[962] S.Ákos2007-12-30 12:06:40

Legyen az egyszerűség kedvéért \overline{BC}=1 és 2CBA\angle=CAB\angle=2\alpha; valamint x:=\overline{AC} y:=\overline{BD}

A szinusztétel értelmében

\frac{2\cos\alpha\sin\alpha}1=\frac{\sin\alpha}{x}

Innét x=\frac 1{2\cos\alpha}. Mivel 3\alpha<180o, ezért BDC derékszögű hsz mindig létezik, így felírható: y=\frac{\overline{BC}}{\cos\alpha}=\frac{1}{\cos\alpha} így a keresett \frac xy arány \frac xy=\frac{\frac 1{2\cos\alpha}}{\frac{1}{\cos\alpha}}=\frac12 (Remélem nem néztem el semmit)

Előzmény: [961] BohnerGéza, 2007-12-30 01:01:09
[961] BohnerGéza2007-12-30 01:01:09

A komáromi Selye János Magyar Tannyelvű Gimnázium Cornides István Emlékversenyének ( 2007.12.07 ) egy szép feladatát ajánlom:

128. feladat: Az ABC hegyesszögű háromszögben az A-nál lévő szög a B-nél lévő kétszerese. A C-ben a BC-re állított merőleges AB-t D-ben metszi. Mennyi az AC / BD arány?

[960] BohnerGéza2007-12-15 14:51:59
Előzmény: [959] Cogito, 2007-12-14 17:02:50
[959] Cogito2007-12-14 17:02:50

Kedves HoA!

A feladatot pár napja én is megoldottam, csak az idő hiányzott, hogy letisztázva közölhető állapotba hozzam. Egyetértek azzal, hogy ez a kör a keresett mértani hely abban az esetben, ha a feladatot az ABC síkra szűkítjük. A részleteket most mellőzve nekem az jött ki, hogy a mértani helynek eleget tévő P pontokra teljesül, hogy

\vec{PC} \cdot \vec{PD}=0 (1)

, ahol a C pontnak O-ra való középpontos tükörképe D. Mint látható, itt (egyrészt) azon pontok halmazáról van szó, melyekből a CD szakasz derékszög alatt látszik. Ez pedig az a C és D pontok nélküli gömbfelület, amely az Általad kapott kör CD körüli megforgatásával áll elő. Itt is igaz (másrészt) hogy a C és D pont is a mértani helyhez tartozik, hiszen a P\equivC, vagy P\equivD esetben egy-egy nullvektor miatt teljesül (1). A levezetés itt is megfordítható, tehát a teljes gömbfelület a keresett mértani hely.

Ez az általános megoldás, hiszen a feladat szövege megengedi, hogy P-t térbeli pontnak tekintsük, a levezetés(ek) pedig ennek az értelmezésnek is eleget tesznek.

Előzmény: [958] HoA, 2007-12-14 08:26:03
[958] HoA2007-12-14 08:26:03

Hát ha senki... Legyen az AB szakasz felezőpontja O, \vec{OA} ={\bf t} , \vec{OC} = {\bf c} . Ekkor \vec{OB} ={\bf -t} , \vec{CA} = \vec{OA} - \vec{OC} = {\bf t-c} , \vec{CB} = \vec{OB} - \vec{OC} = {\bf -t-c} , \vec{CA} \cdot \vec{CB} = ( {\bf t-c} ) \cdot ( {\bf -t-c} ) = ({\bf c-t})\cdot ( {\bf c+t} ) = {\bf c^2 - t^2 } . Legyen \vec{OP} ={\bf p}. Ekkor az előzőhöz hasonlóan \vec{PA} \cdot \vec{PB} = {\bf p^2 - t^2 } A feltételi egyenlet szerint p2=c2, a keresett P pontok tehát az O körüli |c| sugarú körön helyezkednek el, és mivel a műveletek megfordíthatók, a kör minden pontja megfelel.

Előzmény: [957] BohnerGéza, 2007-12-09 23:40:09
[957] BohnerGéza2007-12-09 23:40:09
[956] szegeddiák2007-11-28 21:32:57

Sziasztok...segítségre lenne szükségem néhány feladatjoz..ha vki meg tudja oldani ezeket kérem rakja fel vagy vegye fel velem a kapcsolatot.Igazán sürgős lenne mert pénteken ebből zh-t írok.köszi előre is

tehát a feladatok: kocka élén áll ábrázolni Monge féle ábrázolással csúcsán álló , élén álló oktaéder ábrázolása monge féle ábrázolással teraéder lapján , csúcsán és élén áll szintén monge ábrázolással.

[955] HoA2007-11-28 10:19:55

Úgy látom nem érdekelt senkit, vagy túl egyszerű volt? A súlyvonal tulajdonságából következik, hogy AM=2.A'O=2.R.cos\alpha R = \frac{AM}{2 \cdot cos \alpha} AM hossza és \alpha tehát R-et meghatározza, így a keresett kör középpontok az A körüli R sugarú körön vannak.

Ha O-t és az MA irányt rögzítjük, a megfelelő háromszögek A csúcsába mutató sugár vektorok \phi irányszögére - az MA = A'O iránytól mérve

\alpha-\pi<\phi<\pi-\alpha , az ábrán barnával jelölt CAB körív.

Az MA szakaszt rögzítve a CAB körív minden P pontjába mutató OP vektornak megfelel egy vele ellentétes irányú , a lehtséges Q körülírt kör középpontokba mutató AQ vektor. Így a feladat megoldását az A körüli R sugarú kör azon pontjai adják, melyeknek irányszöge A-ból ( ismét MA irányhoz mérve )

\alpha<\phi<2.\pi-\alpha ( lila körív )

Előzmény: [944] bohmajster, 2007-11-24 13:12:45
[954] Hajba Károly2007-11-28 08:11:00

Még mindig kevés az infó. A négyzet melyik egyenese az egyenes és melyik pontja a pont? Mikorra kellene? Péntekre tudnám elkészíteni a segédábrát, de majd az 'ábrázoló geometriában', oda való.

Előzmény: [953] tyotyke, 2007-11-28 07:52:38
[953] tyotyke2007-11-28 07:52:38

Szia! Igen elhamarkodottan írtam le a dolgokat.Egy pont és egy egyenes két képéről van szó, és ebből kell egy kockát szerkesztenünk, csak a négyzetet azért irtam, mert az már 2-es és onnantól kezdve a magasságvonalak szerkesztésével a kocka csak néhány lépés. Fel kell vennünk egy első fővonalat......stb., leforgatotott háromszögből berajzoljuk a valódi kocka oldalhosszát.....Ezek lépések a szerkesztésből. Remélem igy már érthető!!!:) Köszönöm a segítséget,érdeklődést! Üdv!

[952] HoA2007-11-27 18:47:11

Szia! Biztos vagy te abban, hogy ez két egyenes és két pont a síkban? Nem egy ábrázoló feladatról van szó és egy pont és egy egyenes két vetületét látjuk? ( Ehhez javasolnám az "ábrázoló geometria" témakört ). És mi a feladat? Hogy kell egy (vagy két?) pontból és egyenesből négyzetet és kockát szerkeszteni? Az ábrázoló esetben el tudok képzelni olyan feladatot, hogy szerkesszünk négyzetet, melynek egyik csúcsa az adott pont, oldalegyenese/átlóegyenese az adott egyenes. Vagy szerkesszünk kockát, melynek egyik csúcsa az adott pont, egyik élének/lapátlójának/testátlójának egyenese az adott egyenes. Ezek egyike a feladat?

Előzmény: [950] tyotyke, 2007-11-27 12:18:55
[951] tyotyke2007-11-27 12:22:12
[950] tyotyke2007-11-27 12:18:55

Sziasztok! Végig néztem a forum hozzászólásokat és meg kell hogy állapítsam, hogy itt profik társalognak! Ebből adódóan jött az ötletem, hogy a Ti segítségeteket fogom kérni és remélem segítetek is nekünk! Egy zh feladat megoldására lennénk kiváncsiak, szerkesztővonalak ábrázolásával és némi magyarázattal.A feladat általában egy tetszőlegesen adott két egyenes és két pont a síkban, amiből először egy négyzetet, majd abból egy kockát kell szerkeszteni. Megpróbálom a kiinduló ábrát csatolni, remélem sikerül. Segítségeteket előre is köszönöm!

[949] BohnerGéza2007-11-26 22:40:07

Jogos! Én is mindenkinek ajánlom a végiggondolását!

Előzmény: [948] HoA, 2007-11-26 15:17:06
[948] HoA2007-11-26 15:17:06

Nagyon szép megoldás a "ha derékszögű, akkor..." irányra. Használható-e ez vagy hasonló ábra a másik irányra?

Előzmény: [947] BohnerGéza, 2007-11-26 13:20:09
[947] BohnerGéza2007-11-26 13:20:09

A feladatot a következő érdekes észrevétel miatt tűztem ki:

Legyen ABC C-nél derékszögű. Tükrözzünk a C-n nem átmenő szögfelezőkre! Például Ca' a C-nek az A-hoz tartozó külső szögfelezőre való tükörképe. Ekkor AC egyenes képe AB lesz és BC képe a Ca'-n átmenő AB-re merőleges egyenes, amely érint a b és c indexű hozzáírt köröket. ....

Előzmény: [946] HoA, 2007-11-26 10:29:32
[946] HoA2007-11-26 10:29:32

A másik irány egyszerűbb. Ha a háromszög derékszögű és ismertnek vesszük, hogy a csúcsokból az érintőkörökhöz húzott érintőszakaszok hossza s, s-a, s-b, s-c , akkor az ábrán pirossal jelölt, érintőszakaszokból és sugarakból álló deltoidok itt négyzetek lesznek, ezért

\varrho=s-c;\varrhoa=s-b;\varrhob=s-a;\varrhoc=s és ezért

\varrho+\varrhoa+\varrhob=s-c+s-b+s-a=3.s-2.s=s=\varrhoc

Előzmény: [943] BohnerGéza, 2007-11-22 18:21:24
[945] Python2007-11-25 12:32:17

Legyenek a háromszög oldalai a, b, c, beírt körésnek sugara r, a hozzáírt körök sugara ra, rb, rc (pl. ra az a oldalhoz írt kör) ! Tegyük fel hogy pl. rc=r+ra+rb! Felhasználva hogy a t háromszögterületre 2t=r(a+b+c)=ra(-a+b+c)=rb(a-b+c)=rc(a+b-c)

\frac{2t}{a+b-c}=\frac{2t}{a+b+c}+\frac{2t}{-a+b+c}+\frac{2t}{a-b+c}

A nevezők a háromszög-egyenlőtlenség miatt pozitívak, felszorozva; 2t-vel osztva

(a+b+c)\left[(a-b+c)(-a+b+c)+(a+b+c)(a-b+c+-a+b+c)\right]=

=(a+b+c)(a-b+c)(-a+b+c)

Elvégezve a műveleteket

4c(c2-b2-a2)=0

Itt 4c\neq0, így c2=a2+b2, és ekkor a Pithagorasz-tétel megfordítása miatt a háromszög derékszögű.

Előzmény: [943] BohnerGéza, 2007-11-22 18:21:24
[944] bohmajster2007-11-24 13:12:45

Legyen adott AM szakasz a síkban és \alpha hegyesszög. Határozzuk meg azon ABC háromszögek körülírt körének középpontjainak halmazát, melyek A csúcsánál az \alpha szög található és az M pont az ABC háromszög magasságvonalainak metszéspontja.

[943] BohnerGéza2007-11-22 18:21:24

126. feladat:

[942] Bubóka2007-11-16 17:19:00

Rendi, igyekszem! Köszi!

Előzmény: [939] Hajba Károly, 2007-11-16 15:51:16
[941] Bubóka2007-11-16 17:17:46

NAgyon szépen köszönöm!!!!!! Elnézést, ha nem voltam elég érthető, de abszolute kezdő vagyok itt.

Előzmény: [940] HoA, 2007-11-16 16:20:18
[940] HoA2007-11-16 16:20:18

OK, helyesbítek. Tegyük fel, az adatok most már világosak. A feladat tehát így szól

1) "Bizonyítsuk be, hogy nem szerkeszthető meg a hármszög, ha adott \alpha=\pi/2, ma=1 és fb=2" , illetve

2) "Bizonyítsuk be, hogy nem szerkeszthető meg a hármszög, ha adott a=1 , ma=1 és fb=1"

Ha így van, akkor 2) megoldásának egyik lehetséges módja: Az ábra szerint az A csúcs x koordinátájával (a) fejezzük ki x-et és y-t annak alapján, hogy

x2+y2=1 és \frac{1-a}{1} = \frac{1-x}{y}

Ha jól számoltam, x = \frac{1-(1-a)^2}{1+(1-a)^2} Ezután felírjuk, hogy a felezett szöggel szemközti oldalnak a szögfelező által elvágott két darabja - és így ezek x irányú vetületei is - úgy aránylanak, mint a közrezáró oldalak hossza:

\frac{1}{\sqrt{1+a^2}} = \frac{1-x}{x-a} , persze x-et a-val kifejezve, kapunk a-ra egy magasabbfokú egyenletet. Erről kell (lehet?) megmutatni, hogy a megoldás nincs benne az adott adatokból szerkesztéssel elérhető számtestben.

Előzmény: [938] Bubóka, 2007-11-16 14:03:32
[939] Hajba Károly2007-11-16 15:51:16

Balra fenn van 5 okker gomb és a középsőben TEX tanfolyam. Tanulmányozd, ami mögötte van és fogsz tudni itt is szerkesztett szöveget beírni.

Előzmény: [938] Bubóka, 2007-11-16 14:03:32
[938] Bubóka2007-11-16 14:03:32

Valószínű nem a felfogásoddal van baj! A feladat az, az amit leírtam a 935 alatt, de sajna csak ennyi:

Bizonyítsuk be, hogy az alábbi háromszögszerkesztési feladatok nem szerkeszthetők euklidészi értelemben! A harmadfokú problémáknál vizsgáljuk, hogy megoldható-e szögharmadoló eszközzel. .... és az adatok.

Nem szerkesztésről van szó. Bocsánat, de nem tudok itt egyszerű szövegen kívűl mást "szerkeszteni", ha pedig vágólapról akarok másolni, akkor nem másolja ugyanazt.

Előzmény: [937] HoA, 2007-11-16 09:26:03
[937] HoA2007-11-16 09:26:03

Lehet, hogy kicsit nehéz a felfogásom, de nekem még így sem világos. Légy szíves írd le magyarul, mi a két feladat, valahogy így:

1) Szerkesszünk háromszöget, ha adott \alpha=\pi/2, ma=1 és fb=2

2) Szerkesszünk háromszöget, ha adott a=1,ma=1 és fb=1

Előzmény: [936] Bubóka, 2007-11-16 06:59:56

  [1]    [2]    [3]    [4]    [5]    [6]    [7]    [8]    [9]    [10]